人人范文网 其他范文

gct英语范文(精选多篇)

发布时间:2022-08-09 15:02:49 来源:其他范文 收藏本文 下载本文 手机版

推荐第1篇:GCT英语核心词汇

1abandonvt.放弃,抛弃 n.放任,放纵否定。特别注意放任,放纵的含义2acceiblea.易接近的;可被利用的;易受影响的;可理解的褒义词,通常表达作者对某事物的态度。

3accommodationn.住处,膳宿主要是提供便利,满足需要的意思

4acutea.敏锐的;剧烈的褒义

5additionala.另外的,附加的,额外的通常用这个词引出对上下文观点和事物的追加叙述,因为是顺承关系,又是进一步阐述,引出的下文可以不太关注。

6adequatea.适当的,足够的绝对地反映作者的正态度,表达作者对某观点、某事物的褒扬。

7admitteda.被承认的,被确认无疑的正态度词。

8advocaten.提倡者,鼓吹者vt.提倡,鼓吹对某原因或者提议进行辩护,在阅读中引申为拥护、支持的意思。

9alterv.改变这个改变如果发生在观点上,非常值得关注,因为观点是顺承还是转折对整个文章结构至关重要,例:alter accepted views of ancient literary works10amuseda.愉快的,好玩的,开心的通常为褒义词

11appropriatea.适当的正态度

12approvaln.赞成;承认;正式批准不是停留在许可的表面,表达了赞同、认可、嘉奖的正态度

13arguableadv.可论证地,可争辩地贬义

14aertv.断言,声称没有正负态度的倾向性,提出观点、理论

15aertionn.主张;断言,声明确认的观点

16aevt.评定,评估 提出观点

17aociatevt.使发生联系,使联合产生联系

18attackvi.攻击n.进攻,攻击这个词常被用于提出与上文对立的观点。19attributen.属性,品质表示因果关系

20availablea.可用到的,可利用的比较明显的褒义词,当然反映作者的正态度,对于这个词本身及其修饰的主语一定要给予强烈关注。

21backgroundn.背景,后台有个别文章的第一段并没有直指focus,而是作为一种引子,引出下文,这样的段落常被问及作用,它们的作用就是提供了一种background22barriern.障碍物,栅栏,屏障这个词常常引出事物的缺陷,因为缺陷是必然考点,引出缺陷的这个词也就当然值得关注了

23besidesadv.此外prep.除…之外注意,besides有together with的含义,它虽然用于引出新事物,但是却暗含新事物和上文事物有着同样的特征,即上下文所述观点一致,是顺承关系的叙述

24biasn.偏见,偏爱否定的态度

25blamen.过失,责备 vt.责备,谴责表达事物缺陷的特征词

26ceasev.停止,终了

27challengen.挑战 vt.向…挑战在阅读中,这个词用得更多的是表达了一个新事物对旧事物得背叛和挑战,表达了新旧观点的对立。

28chargen.负荷;费用;掌管vt.控诉;责令,告戒;指示;收费

29chiefa.主要的,首要的,首席的;主任的 n.首领,领袖最高级的特征词30coexistvi.共存

31coherenta.粘在一起的;一致的,连贯的

32collidevi.碰撞,抵触相反观点

33communityn.社区,团体,大众常表示一个生物群落或人居群体

34compensatev.偿还,补偿,付报酬

35competitionn.竞争,竞赛

36complicatev.(使)变复杂

37componentn.成分 a.组成的,构成的

38composev.组成;写作

39concernvt.涉及,关系到

40condemnvt.声讨,谴责;判刑这是一个常在态度题中出现的单词,表达负态度

41confirmvt.确定,批准;使巩固;使有效这个词关系到一个观点、证据是否有效,如果一个观点被confirm了,那的确是件欢欣鼓舞的事情,因为它必然是作者所支持的观点

42conflictn.斗争,冲突vi.抵触,冲突常常用以引出对立的观点,表达事物之间的对立关系。

43confrontvt.使面临;对抗这个词常常用以引出对立的观点,也同时表达了一种贬义的色彩,就是遭遇到了不好的事情。

44confuseda.困惑的,烦恼的负态度词

45consistenta.一致的,调和的;坚固的正态度

46contemporaryn.同时代的人a.当代的;同时代的比较

47contendv.斗争,竞争;主张这个词常用以引出对立观点

48contestn.争论;竞赛v.争论,争辩;竞赛提出观点

49contradictvt.反驳;反对;否认;与…矛盾;与…抵触;与…相反

1) 2) 3) 50To aert or expre the opposite of (a statement).驳斥一种说法; To deny the statement of 否认…得说法; To be contrary to; be inconsistent with 与…相矛盾;与…抵触 conventionala.惯例的,常规的;习俗的,传统的传统的、惯例的通常是作者反对的,要屏弃的,所以这个词引出的观点属于旧观点,之后必然有新观点取代它。51crashn.碰撞;坠落,坠毁v.碰撞;坠落,坠毁贬义

52criticala.批评的,评论的,鉴定的;危急的;临界的常在态度题中出现,表达作者的负态度

53criticizev.批评,责备表达一种负态度

54debatev.争论,辩论 n.争论,辩论这是个值得关注的单词,因为它的出现往往意味着有对立的观点,而观点的对立关系往往是考察的重点。

55defectn.过失,缺点表示事物缺陷的特征词,如果这个词出现,后文值得关注,因为缺陷是一个重要的考点,表示缺陷的其他特征词:shortcoming, imperfection ,deficiency

56definitea.明确的,一定的正态度

57demonstratea.可论证的这个词如果在阅读中出现,很值得关注,因为一个可被证明的观点是有效的观点,有效的观点也就是作者支持的观点capable of being demonstrated

58depreionn.沮丧,消沉;低气压,低压贬义态度

59descendv.下去vi.下来,下降下降表示的是一种变化,因为变化在阅读中常常成为考点,所以出现这个词的值得关注。

60destructivea.破坏(性)的在阅读中看到这个词应该首先想到它是个贬

义词,有时候也引出某事物的缺陷,成为考点。

61discouraginga.令人气馁的贬义词

62distincta.清楚的,明显的;截然不同的,独特的作为差别讲时,是个很值得关注的词,因为差别经常成为考点

63drawbackn.缺点,障碍;退换的关税,退税(指进口货物在出口时退还其进口时的关税)缺陷的特征词,值得强烈关注,因为缺陷经常成为考点

64eliminatevt.排除,消除表示彻底消失了,为考点

65embracevt.拥抱;包含;收买;信奉 vi.拥抱n.拥抱正态度词,表达欢迎,赞同

66enforcen.执行,强制

67enhancevt.提高,增强表达事物的变化,值得关注

68evolutionn.进展,发展;演变,进化达尔文的进化论,在阅读中出现通常遭到作者反对

69exaggeratev.夸大,夸张贬义词,时常引出事物的缺陷,值得关注。同义词有:overstate, overemphasize

70fruitfula.果实结的多的,多产的;富有成效的褒义词

71genuinea.真实的,真正的;诚恳的褒义词

72identifyvt.识别,鉴别;把…和…看成一样 v.确定当含义为to be or become the same 的时候表达两种事物的关系,常常成为考点,值得关注

73imitatevt.模仿,仿效;仿制,仿造表示两者有联系

74impactn.碰撞,冲突,冲击;影响,效果结果类命题的标志词

75impropera.不适当的,不合适的;不正确的;不合理的,不适宜的事物缺陷的特征词,值得关注

76inaccuracyn.错误事物缺陷的特征词,值得关注

77incompletea.不完全的,不完善的负态度词,事物缺陷的特征词,值得强烈关注

78increasevt.增加,加大vi.增加;繁殖 n.增加,增大,增长表示事物变化的词,要予以足够关注,最好在读文章的时候给出标记,例如用一个向上的箭头表示变化的趋势

79incrediblea.难以置信的表示惊奇的态度

80indifferencen.不关心通常出现在\"态度题\"中,充当混淆项

81inefficiencyn.无效;无能;不称职负态度词,事物缺陷的特征词,值得强烈关注

82initiala.最初的,初始的;词首的 n.词首大写字母旧观点的特征词83innovativea.创新的,革新(主义)的新事物,新观点的特征词,新的总是作者支持的

84insightn.洞察力,见识 正态度词

85insufficiencyn.不足负态度词,事物缺陷的特征词,值得强烈关注

86interpretv.解释,说明;口译,通译;认为是…的意思这个词出现,它能否有效地解释观点、现象是个关键,也是应该关注的焦点

87invalidn.病人,在、残废者负态度词

88investigatev.调查,研究常常作为说明观点的论据,一般来说,调查的具体内容不必细看,关键是要看这个调查是否能有效支持证据

89justifieda.正当的;合法的;合理的褒义词

90meritn.优点;价值 v.有益于褒义词

91misunderstandvt.误解,误会

92modesta.谦虚的,谦让的;适度的表示一种适度的态度或情绪,常出现在态度题的正确选项里,和一个有感情色彩的词连用,例如:modest commend,适度的赞扬,说得严谨客观,比较容易成为正确答案,如果modest所修饰的主态度词所表达感情色彩正确,则是正确答案无疑。

93negativen.否定;负数;底片 a.否定的;消极的;负的;阴性的 vt.否定;拒绝(接受)贬义词

94neglectvt.忽视;疏忽;漏做 n.忽视;疏忽;漏做忽略了本应该在意的东西,本应该予以关注的东西,所以这个词本身就说明,它忽略的对象读者应该予以足够的关注,统计表明,这的确常成为考点;同时\"忽略\"是个缺点,容易被考,值得关注。95normallyadv.正常地,通常地这本来是个很简单的词,但是在GRE阅读中却有不同寻常的意义,因为\"通常的\"观点在阅读中总是成为旧观点和作者要屏弃的观点,ETS喜欢标新立异,对于多数人赞同的、通常的、传统的东西多半是要反对的

96noveln.小说,长篇故事 a.新奇的,新颖的,异常的当novel作为\"新的\"讲的时候,通常是作者支持、赞同的观点,也通常是作者想大篇幅叙述的观点,所以值得予以强烈关注

97objectiven.目标,目的 a.客观的经常作为态度词出现

98opposeda.反对的,敌对的负态度词

99oppositea.相对的,对面的;对立的,相反的;对等的,对应的 n.相反的事物用以引发对立的观点,值得关注

100originaln.原物,原作 a.最初的;原始的;独创的,新颖的一个含义是\"原作\";第二个含义\"新颖的,新奇的\",因为是新的,故通常是作者赞同的

101outlinen.大纲,轮廓,略图;外形,要点,概要 vt.描画轮廓;略述是整体的结构词

102outstandinga.突出的,显著的褒义词

103outweighv.在重量(或价值等)上超过常用以比较两个事物的重要程度,涉及到常考查的事物之间的关系,值得关注

104overestimatevt.评价过高 n.估计的过高,评价的过高表示了缺陷、过失,经常被考,值得关注

105overlookvt.俯瞰;耸出;远眺;没注意到 n.眺望;俯瞰中的景色在阅读中值得关注的含义是\"忽略\",即to look past, mi, ignore

106overwhelmvt.淹没;覆没;制服;压倒表示\"占绝对优势、绝对重量的\",例如:overwhelming majority,压倒性的多数

107partiala.部分的,局部的;偏袒的,偏爱的

108paivea.被动的

109persuasivea.有说服力的褒义词,通常用于作者赞同的观点及论据上,值得关注

110peimismn.悲观,悲观主义

111positivea.肯定的;实际的;积极的;绝对的;确实的正态度词112preciousa.宝贵的,贵重的,珍爱的;过于精致的褒义词

113precisea.精确的,准确的褒义词

114predominanta.卓越的,支配的,主要的,突出的,有影响的褒义词115preferencen.偏爱;优先选择比较的含义

116prejudicen.偏见,成见;损害,侵害 v.损害常成为考点的表缺陷的词117prevailvi.流行,盛行;获胜,成功褒义词

118previousa.在前的,早先的 adv.在…以前这个词常用来引出旧观点,也就是作者要屏弃的观点

119primitivea.原始的,远古的;粗糙的,简单的这个词有三个基本含义,当它为\"早期的\"讲的时候通常用以引出旧观点

120profoundlyadv.深刻地,忠心的几个含义中,\"深刻地\"用的最多,褒义词,即having intellectual depth and insight

121prohibitionn.禁止,阻止;

122promptlyadv.敏捷地,迅速地

123properlyadv.适当地,完全地褒义词

124protestn.主张,断言;抗议常用以引发对立的观点,值得关注

125puzzlen.难题,谜 v.(使)迷惑,(使)为难常为\"解释针对问题\"类型的文章的特征词,尤其是当它出现在首段的时候,很可能意味着下文都在寻求对它的正确解释

126recruitn.新兵,新分子,新会员 vt.使恢复;补充;征募 vi.征募新兵;复原有\"新\"的含义,故作者通常给它正态度

127refineda.精制的;优雅的;精确的褒义词

128reformn.改革,改善,改良运动,感化 vt.改革,革新,重新组成 vi.革新,改过,重组改良、革新都是作者一贯支持的观点

129reinforcevt.加强,增援;补充,增加…的数量;修补,加固 vi.求援,得到增援n.加固物是个表示事物之间的关系的词汇,值得关注,因为事物之间的关系常常成为考点

130rejectn.被拒之人,被弃之物;不合格品;落选者;不及格者vt.拒绝;抵制表示否定了某事物、某观点

131relationn.关系,联系;叙述,故事;亲戚这个词汇在阅读中出现要予以足够的关注,因为事物之间的关系常常成为考点,而relation将引出对事物关系的直接叙述

132reliablea.可靠的,可信赖的褒义词

133removaln.移动;免职;切除否定了某事物

134resemblevt.象,类似这个词指的是两者相似,因为反映了事物之间的关系,常常成为考点,所以值得关注。在阅读中,相似和等于几乎可以替换,因为说相似着眼点只在共同之处,对不同和差异一般没有论述。

135restatevt.重新叙述;重申有\"新\"的含义,重申的理论、观点通常都是作者支持的

136revisevt.修订,校订,修改修订多半是作者支持的,因为有推陈出\"新\"的意思

137revolutionn.革命;旋转这个词的英文解释的核心是change,因为有革\"新\"的含义,所以通常是作者所支持的

138rivaln.竞争者,对手 v.竞争,对抗,相匹敌这个词所体现的对手不光是单纯的竞争关系,还有衣层隐含含义是相匹敌的,即one that equals another in desired qualities; peer

139routinen.例行公事;常规;日常事物;程序惯例、常规、习俗都有\"旧\"的含义,因此一般是作者将要屏弃的旧观点

140sanctionn.批准;同意;支持;制裁;认可 v.批准;同意;支持;鼓励;认可正态度词

141schemev.计划,设计;图谋,策划 n.安排配置;计划;阴谋;方案,图解,摘要

142segmentn.段,节,片段 v.分割是部分的标志词。

143shortcomingn.缺点,短处事物缺陷的特征词,值得予以强烈关注,同义

词:deficiency, defect

144similarityn.类似,类似处一般用以表达事物之间的关系,经常成为考点,值得关注

145statisticsn.统计学;统计表在阅读中这个词出现,通常引出一段论据用以证明某观点,这个统计的具体内容、方式、数据等不必关注,它能否有效的支持其要说明的观点是关键,也是读者应当关注的焦点

146subjectvt.使屈从于…,使隶属n.主题,题目意思为从属的时候表达了两个事物之间的关系,容易哼为考点,值得关注

147subordinatea.次要的,从属的,下级的 n.下属 v.服从

148substituteda.被取代的,被代替的表达式之间的关系,常成为考点,值得关注

149succeivea.连续的注意与\"成功的\"相区别

150surveyvi.测量土地vt.调查看到这个词立刻想到它是论述某观点的论据,其内容并不重要,重要的是这个证据能否证明某观点有效。

151suspectn.嫌疑犯 a.令人怀疑的,不可信的负态度词

152tediousa.单调乏味的,沉闷的,冗长乏味的贬义词

153tentativen.试验;假设 a.实验性的,试探的,尝试的;暂定的它是论述某观点的论据,其内容并不重要,重要的是这个证据能否证明某观点有效

154transitionn.转变,转换;表示事物变化的词,值得关注

155unavailablea.难以获得的贬义词,反映事物的缺陷,值得关注

156underestimatevt.低估,看轻贬义词

157underminev.破坏贬义词,表达对某事物、观点的否定

158undetermineda.未确定的;未解决的这个词出现有两种可能,一种是用以说明前面观点的无效性,另一种是引发下文的解决方案,如果是这种情形,读到这个词,读者就要开始寻找了

159uniforma.统一的,相同的,一致的;始终如一的, n.制服表示事物之间的关系,在遇到这个词的时候,最好用=给个标记

160uniquea.唯一的,独特的表明最高级的词汇,当它在阅读中出现,最好做个标记,因为最高级常常能帮助判断正确选项的正误,但这个词并不总代表惟一,有时候也是\"独特的\"的含义,要注意分辨。例:they can make a unique contribution(特殊贡献)to society.

161universala.普遍的,全体的,通用的因为有\"普遍的,通用的\"含义,这个词常常引出一个旧观点,也就是作者通常反对的观点。

162unrealistica.不切实际的负态度词

163vaguea.含糊的,不清楚的负态度词

164validatevt.使有效,使生效;确认,证实,验证是个证明某观点有效性的词,看到这个词,对它所支持的观点要予以足够关注,因为那一定是作者支持的观点165varietyn.变化,多样性;品种,种类变化和不同常常成为考点,所以这组词无论谁出现,都值得关注

166verifyvt.检验,校验,查证,核实是个证明某观点有效性的词,看到这个词,对它所持的观点要予以足够关注,因为那一定是作者支持的观点

167violatevt.违犯;冒犯;干扰引出对立观点,值得予以足够关注168whereasconj.然而,反之;鉴于;尽管,但是转折关系词

169whileconj.当…的时候;虽然 这是一个表达事物之间对比关系的连词,所引导的句子很值得关注。

推荐第2篇:GCT研究生复试英语自我介绍

研究生复试英语自我介绍

1.

开场白

Good morning/afternoon.I am very glad to be here for this interview.

2.

姓名,年龄,家乡,毕业院校、专业及院系

First let me introduce myself.My name is ZhangHaitao, I am 23 years old.I come from Xinxiang Henan province, a very beautiful modernization city.I will finish my undergraduate education in Shangqiu Normal University in June, Majoring in Surveying and Mapping Engineering in the Department of Environment and Planning, and now, I am trying my best for obtaining a key to China University of Geosciences.

3.

性格,爱好,实践经验

Generally speaking, I am a hard working student especially do the things I am interested in.I will try my best to finish it no matter how difficult it is.I am open-minded, willing and have broad interests like basketball, tennis, Chinese che and reading, especially in dealing photos with Photoshop.For example, during the past four years, I have accomplished several pictures: one is for the dinner’s background of our department, and the other is to attend our university’s examination.Furthermore, I am interested in software programming with Visual Basic and have paed The Grade two of The National Computer Rank Examination.Well, in my spare time, I often go to English corner to practice my oral English on every Thursday, and write compositions to improve my written ability.But I know my English is not good enough, I will continue studying.In Julyin the last year, I finished my graduate project with The Technology Design and Implementation of Photogrammetry Field Work, which Based on the status of the development of photogrammetry, taking into account the outcome of lots of research scholars of years, centering the photograph control surveying, control points selecting and barbing, tone painting and interpreting of aerial photography and field work methods based on GPS joint measurements job content and requirements, the paper systematically expounded the related basic concepts, basic theory, basic requirements of the field work of photogrammetry technical design, the methods of cope with aerial photography, as well as dealing with the basic methods topographic map production requirements.In addition, I have also finished some projects about embeddedsystem by using MCU when I was a junior.

4.为什么想读研,将来愿意从事的方向,读研时的打算

Although I have broad interests in many aspects and grasp the eential knowledge of the major, but I think at present, I can do many things just in a superficial level, but not be competent

to do things profeionally owing to lack of ample knowledge and ability.SoI think further study is still urgent for me to realize self-value.

推荐第3篇:GCT英语模拟试题训练

2013年GCT英语模拟试题训练

GCT考生要想在GCT英语考试中获得高分,模拟试题的训练必不可少,在模拟试题的训练中不仅可以熟悉并掌握更多的词汇量,最重要的是GCT英语能力能得到一个有效的提升。

1.As a matter of rule, the scrap value of a veel can hardly be at ________ with the sound value.

A.par

B.bearish

C.collapse

D.rally

2.As a defense against air-pollution damage, many plants and animals_________ a substance to absorb harmful chemicals.

A.relieve

B.release

C.dismi

D.disclose

3.As a salesman, he works on a (an) ________ basis, taking 10% of everything he sells.

A.income

B.commiion

C.salary

D.pension

4.As all of us know, color-blind people often find it difficult to ________ between blue and green.

A.separate

B.distinguish

C.compare

D.contrast

5.As everyone knows, the exchange ________ fluctuates almost daily.

A.ration

B.ratio

C.rate

D.interest

答案解析

1.[答案]A

[翻译]按照常规,报废船只的价值是比不上好船的价值的。

[分析] par 的意思是“等价”;bearish的意思是“看跌的”;collapse的意思是“暴跌”;rally的意思是“回升”。

2.答案]B

[翻译]作为一种防止受到空气污染的损害的手段,许多动物和植物释放一种吸收有害化合物的物质。

[分析]release的意思是“释放,解除”;relieve的意思是“减轻,安慰,解除”;dismi的意思是“解雇,开除”;disclose的意思是“揭发,揭露”。

3.[答案]B

[翻译]作为一个销售员,他按照销售量的10%提取佣金。

[分析] commiion的意思是“佣金,回扣”它是一个经济词语;income 的意思是“收入,所得”;salary“(按月领取的)薪水”; pension的意思是“养老金,退休金”。

4.[答案]B

[翻译]众所周知,患有色盲症的人很难区别蓝色和绿色。

[分析] distinguish的意思是“别,区别”,经常用于distinguish one thing from the other 和distinguish between A and B结构中;separate的意思是“分离,分开,隔离”,常和from连用;compare的意思是“比较,比喻”,可用于compare„with„结构中,意思是“与„„相比”;contrast 的意思是“(使)对比,(使)对照”,常用在“in contrast to”的结构中,意为“与„„相对照”

5.[答案]C

[翻译]正如人们所知,兑换率几乎每天都波动。

[分析]rate的意思是“比率;速度”;exchange rate是固定的搭配,意思是“兑换率”;ratio的意思是“定量配给”;ratio的意思是“比,比率”,表示两个量之间的关系。例如:Men outnumber women here in the ratio of three to one.此地男子数量以三比一超过女子。The ratio of 1 to 5 and 20 to 100 are the same.1与5之比和20与100之比相同。interest的意思是“利息”。

资料来源:环球卓越

推荐第4篇:GCT考试英语大纲(版)

本人花费时间整理的最有价值GCT资料,完全免费下载,如果您觉得资料不错,请记得给我评价,您也可

以获得积分哟。谢谢!

一、考试目的

外语运用能力测试旨在考察考生目前所具备的实际英语水平、阅读英语能力和运用英语能力。通过英语词汇量、语法、阅读、理解、日常口语等内容的测试,了解考生使用英语的综合能力。

二、试题结构 1. 题型与题量

本部分共有50道题,考试时间为45分钟。试卷包括语法和词汇、阅读理解、完型填空、会话技能四部分 (1)语法和词汇

共有10道题,每题2分钟。前5道题为词汇题,后5道题为语法概念题。该部分时间大约为8分钟,分值为20分。 (2)阅读理解

共有4段独立的短文,每段短文约有150个英语单词。短文内容涉及政治、经济、历史、地理、文化、科技、人文、时事等。要求考生阅读每段短文后,回答5个问题。每个问题后有4个答案选项,其中1个选项为正确答案,要求考生选出正确答案。4段短文中,1篇较易,2篇难度适中,1篇较难。从近几年的真题来看,每年阅读理解最后一篇文章为应用文。该部分考试时间大约为21分钟,分值为40分。 (3)完型填空

提供一篇约240个单词的短文,在短文中有10个空白。每个空白有4个填空选项,其中1个为正确答案,要求考生选出正确答案。该部分考试时间大约为8分钟,分值为20分。 (4)会话技能

有10段英语简短对话,每段对话是不完整的,在每段对话后有4个答案选项,要求考生从中选出1个最符合情景和习惯用法的答案,使得整个对话通顺完整。该部分考试时间大约为8分钟,分值为20分。 2. 试题难易程度

试题由难分为3个等级,每套试题中容易、一般和较难的题目比例为3:5:2。 3. 试题评分标准

本部分试题满分为100分,每道题2分。考生须从每道题所列的A、B、C、D四个备选答案中选出一个正确答案,多选、不选或错选均不得分。

三、命题范围

(1)英语使用能力考试命题范围的水平相当于四年制大学非英语专业毕业生应达到的水平。着重测试考生运用英语的能力,要求考生熟练掌握4000个以上的英语词汇和基本语法内容。考生需具有一定的阅读、写作和口头会话的能力,并具有一定的社会、生活和工作背景知识。(1)词汇和语法部分主要是测试考生的基本词汇和语法规则。

(2)阅读理解部分主要测试考生英语的阅读、理解、分析、判断能力。考生需要掌握一定的词汇量和具有一定的阅读速度以及知识背景。4段短文中,其中1段为科普内容,1段为英语应用文。

(3)完型填空主要测试考生对词汇和语法的掌握程度,语法内容包括冠词、名词、代词、形容词、副词、介词、连词、分词、不定式、从句、时态、语态、倒装和虚拟等。同时,考生还需要具有一定的阅读理解能力。词汇与语法的比例为1:2。

(4)会话技能主要测试考生使用英语进行日常会话的能力,这些会话涉及的是生活中的常见情景,测试考生理解会话的情景、把握对话人的角色以及掌握英语口语习惯用法的能力。

推荐第5篇:GCT考试英语完形填空专项练习题(A)

2013年GCT考试英语完形填空专项练习题(A) 来源:中国在职教育网www.daodoc.com 第一题:

Long ago there was a poor farmer called Fred.Fred and his wife, Doris lived 1 together in their small old house.One winter night, the Luck Fairy (仙女) visited them .

“Fred, you’re a 2 farmer.I’d like to give you a wish,” said the Luck Fairy.

“A wish?” Said Fred.

Fred and Doris smiled at each other.Then Fred said, “ Thank you , Luck Fairy.We’re very 3 and happy.”

“ 4 we’re old, we still work in the field every day,” said Doris.

“You wok very hard but you 5 very little money.Would you like some gold coins ” asked the Luck Fairy.

“Oh no , my dear Luck Fairy.We’re poor.But we have 6 food to eat.” Replied Fred.

“You can use the gold coin to buy some clothes.The winter here is very cold,” said Luck Fairy.

“Though we haven’t got 7 clothes, we’ve got enough,” said Doris.

“Well, what about a nice new house?” Asked Luck Fairy.

“Thank you, but I 8 my small old house very much.I’ve lived here since I was born.I don’t 9 a new house,” said Fred.

“You’re quite different from other people.I like you very much,” said the Luck Fairy.“I wish you happine and Luck forever.” Then the Luck Fairy 10 and never came back.

1.A.sadly B.happily C.worried D.anxiously

2.A.bad B.lazy C.good D.unhelpful

3.A.healthy B.careful C.difficult D.important

4.A.If B.But C.Because D.Though

5.A.cost B.lose C.make D.borrow

6.A.no B.little C.enough D.expensive

7.A.old B.many C.bad D.clean

8.A.hate B.love C.need D.dislike

9.A.need B.see C.buy D.build

10.A.smiled B.nodded C.laughed D.disappeared 答案解析:

1.B。根据下文我们知道,这对夫妇生活得很愉快。

2.C。正因为Fred是一个好农夫,仙女才要奖励他。

3.A。比较这四个词的意思不难发现与happy 并列的是healthy。

4.D。根据still可知选though。虽然他们年纪大了,但仍然能够在田里干活。

5.C。make money意思是“赚钱”。

6.C。根据文意,他们对一切都感到知足,包括食物他们也觉得足够吃了。

7.B。他们没有许多衣服,但对他们来说却已经够穿了。

8.B。从后面的句子可知,他是喜欢那个房子的。

9.A。根据上文,他们喜欢自己的小屋,所以不需要新的。

10.D。根据never come back可知仙女消失了。 第二题:

Tom grows the nicest vegetables and fruits and the most beautiful flowers in the village.更多内容请参见:www.daodoc.com www.daodoc.com

Plants grow in Tom’s garden all through the __1__ and they are much __2__.

Tom cuts some flowers for his sitting room table, eats some fruits and vegetables, but he __3__ most of them in the market.His vegetables, fruit and flowers are so __4__ and beautiful that they sold much more __5__ in the market than those of other villagers.

How does Tom grow these beautiful things? He is so __6__ that he just sits under his orange tree with his radio.

He __7__ the music all day.

That is quite true.Tom __8__ things in spring, summer, autumn and winter.After that he sits with his radio.And everything __9__.It is the music that does the work.Tom knows more clearly that music makes the biggest vegetables and the most beautiful flowers.Plants love __10__ as much as people.

1.A.week B.month C.season D.year

2.A.better B.worse C.le D.later

3.A.buys B.sells C.borrows D.lends

4.A.dear B.bad C.big D.small

5.A.politely B.quickly C.slowly D.carefully

6.A.angry B.busy C.tired D.lazy

7.A.listens to B.hears C.watches D.speaks

8.A.fills B.plants C.throws D.makes

9.A.does B.moves C.grows D.plays

10.A.work B.rain C.stories D.music 答案解析

1.D。根据下文汤姆春夏秋冬都会在园子里种各种植物,说明园子里一年到头都长着植物,故year为正确选项。

2.A。很明显,汤姆的植物园比村子里其余人的应好得多,故应用比较级better。

3.B。根据文意,汤姆到市场上去应是卖自己种植的花,水果及蔬菜,故选sells。

4.C。这里应选择一个褒义形容词修饰汤姆的蔬菜水果,故只能选big。

5.B。因为蔬菜水果长得好,所以在市场上卖得很快,故quickly为正确选项。

6.D。根据下文汤姆没有花很多的时间去管理园子而是坐在树下听音乐,故选lazy。

7.A。表示听音乐这一动作应用listen to 。

8.B。根据文意,汤姆春夏秋冬在园子里是在种各种植物,故选plant。

9.C。grows在这里表示园子里的一切在不断地生长。

10.D。上文讲到音乐使得园子里的一切又大又美,说明植物也像人一样喜欢音乐,故选music。 第三题:

Mr.Green was ill and went to the hospital.A doctor __1__ and said, “Well, Mr.Green, you are going to __2__ some injections, and you′ll feel much better.A nurse will come __3__ give you the first one this evening, and then you′ll __4__ get another one tomorrow evening.” __5__ a young nurse came to Mr.Green′s bed and said to him, “I am going to give you your __6__ injection now, Mr.Green.Where do you want it?”

The old man was __7__.He looked at the nurse for a __8__, then he said, “__9__ has ever let me choose that before.Are you really going to let me choose now?”

“Yes, Mr.Green,” the nurse answered.She was in a hurry.“Where do you want it?”

“Well, then,” the old man answered __10__ “I want it in your left arm, please.”

更多内容请参见:www.daodoc.com www.daodoc.com

1.A.looked for him B.looked him over

C.looked after him D.looked him up

2.A.get B.give C.make D.hold

3.A.so B.but C.or D.and

4.A.must B.can C.had better D.have to

5.A.In the morning B.In the afternoon

C.In the end D.In the evening

6.A.first B.one C.two D.second

7.A.confident B.surprised C.full D.hungry

8.A.hour B.minutes C.year D.moment

9.A.Somebody B.Anybody C.Nobody D.people

10.A.with a smile B.in timeC.in surprise D.with tears in his eyes

答案解析:

1.B。look for sb/sth 意为“寻找„„”;look after sb 意为“照料„„”;look up sb意为“看望„„”;而look over sb意为“检查某人”,最贴近文意,为正确选项。

2.A。医生要对格林先生进行注射,格林先生是动作的接受者,故应选get。

3.D。空白部分前面I come 和后面的give形成承接关系,所以应用 and连接。

4.D。must 不可以用于将来时,根据文章意思,应选have to。

5.D。与上文this evening相对应,In the evening应为正确选项。

6.A。one 填入空白部分显得画蛇添足,根据上下文这是第一次注射,应用first。

7.B。老人对护士的提问应感到surprised, 因为下文提到从来没人问过他这样的问题。

8.D。对护士的提问, 老人思考了一会儿, 故应选moment。

9.C。老人感到奇怪, 是因为没有人问过这样的问题, 故应选nobody。

10.A。老人想捉弄一下这个小护士。按常理, 应是带着微笑取笑她, 故应选with a smile。

更多内容请参见:www.daodoc.com www.daodoc.com

推荐第6篇:GCT英语真题及答案

第四部分 外语运用能力测试(英语)A卷(50 题,每小题 2 分,满分 100 分)

Part One Vocabulary and Structure

Directions:

There are ten incomplete sentences in this part.For each sentence there are four choices marked A, B, C and D.Choose the one that best completes the sentence.Mark your answer on the ANSWER SHEET with a single line through the center.

1.If the car you have rented is clearly unsatisfactory, you can always it for another.A.shift B .exchange C .switch D.replace 2.Every manager needs a secretary that he can to take care of something that may occur in his absence.A.bring on B .hold on C .count on D.focus on 3.The shirt is a real bargain because it is good in quality and in price.A.valuable B .remarkable C .available D.reasonable 4.Shopping for cloths is not the same experience for a man it is for a woman.A.when B .that C .as D.than 5.Nina back home if she had known that her husband would go to the bus stop to meet her.A.couldn’t have walked B .shouldn’t have walked C .mustn’t have walked D.wouldn’t have walked 6.A news report is usually very short, when it is about something very important.A.besides B .except C .apart from D.except for 7.In this advanced course, students are required to take performance tests at monthly .A.gaps B .lengths C .distances D.intervals 8.American women were the right to vote until 1920.A.ignored B .refused C .derived D.denied 9.Seldom any mistakes during my past few years of working there.A.would I make B .have I made C .I made D.shall I make 10.The proposal seems to oppose the government economic policy.A.designed B .to design C .have designed D.to have designed Part Two Reading Comprehension Directions:

In this part there are three paages and one chart, each followed by five questions or unfinished statements.For each of them, there are four choices marked A, B, C and D.Choose the best one and mark your answer on the ANSWER SHEET with a single line through the center.

Questions 11-15 are based on the following paage:

Happine is becoming a huge area in psychological research and even in government policy, with the UK government exploring a “ happine index” .It is tough, though, to define exactly what happine is, and what makes us happy.There are two broad ways of looking at happine: short-term happine (a great cookie, a bottle of wine) and long-happine (financial security, achieving your goals).Both types of happine are valid, and important.The problem is, they’re often in competition. Let’s say you’ve got a goal of losing 50 pounds this year.You know you’d be happier and healthier if you weren’t carrying that extra weight.To achieve long-term happine, you need to go on a diet.In the short-term, though, it’s not that easy.A chocolate cake, or a large gla of wine, might just the thing to cheer you up at the end of a long dayOf any kind.

21.According the survey, more than half the people believer that .A.table salt should be used in cooking B .it is neceary to eat sea product C .sea salt is high-sodium product D.sea is a healthier choice than table salt

22.The different between table salt and tea salt lies in that table salt ______.A.contains more natural minerals than sea salt B .forms clumps more easily than sea salt C .taste better than sea salt D.goes through le pro ceing than sea salt

23.The word “ strip” in Para.2 is closest in meaning to “ ” .A.b reak B .add C .remove D.change

24.According to the paage, the proper amount of salt consumption is related to one’s

______.A.appetite B .age C .sex D.weight 25.According to the paage, Americans .

A.c onsume much more salt than they actually need

B .c onsume about 2.300 milligrams of salt a day

C .p refer fresh food to salted proceed food D.try to limit their salt consumption of any kind

Questions 26-30 are based on the following chart:

26.A proper title for the chart is “ ” .A.Schedule Chart for Job Hunting B .Flow Chart for Recruitment Proce C .Descriptive Chart for an Open Position D.Organization Chart for a Hunting Company

27.The chart is probably designed for .A.job applicants B .school graduates C .the company general manager D.the human resources department

28.What happens right after the interview? A.The company will review other interviewees.B .The company will notify who get an offer.C .Th e applicants will confirm job offers with the company.D.The applicants will call in to check whether they get the offer.

29.A work contract is signed when .A.all options have been considered B .there is no more interviewee to review C .the applicant accepts the company’s offer D.the company finishes training for its new staff

30.What does “ Com.” in “Com.Offer” poibly mean? A.Confirm B .Company C .Compensate D.Communicate Part There Cloze Directions There are ten blanks in the following paage.For each numbered blank, there are four choices marked A, B, C and D.Choose the best one and mark your answer on the ANSWER SHEET with a single line through the center.

Where do our favorite foods come from? The truth may __31__ you.Did you know curry(咖喱) isn’t Indian ? Did you know Americans weren’t the __32__ to eat hamburgers? Or did you know pizza wasn’t created in Italy?

First, let’s talk about curry.Many people think the English found out about curry from people in India in the 1600s.But__33__, wealthy English people were cooking with curry spices hundreds of years__34__ British ships traveled to India.In fact, the word “curry” can be found in the English language as far back as 1377.Cooks of wealthy English families created curry dishes, and later these __35__ caught on in other parts of English.

___36___ pizza, this dish was probably first made in Persia (what is now Iran).The Persians were eating round, flat bread with cheese in the 500s – nearly one thousand years before pizza caught on in Naples, Italy!

Finally, let’s look at the truth behind __37__.Many people think hamburgers are an American food.However, _38_some stories, hamburgers came from Hamburg, Germany .A German named Otto Kuasw made the first hamburger in 1891.Four years later, German sailors __39__ hamburgers to Americans.

Where foods come from isn’t nearly as __40__ as how they taste – delicious! So ,go get some of your favorite food and dig in.

31.A.upset B.surprise C.annoy D .inspire 32.A.one B.last C.first D.nation 33.A.in detail B.in shore C.in turn D.in reality 34.A.before B.after C.when D.while 35.A.cooks B.families C.dishes D.places 36.A.Compared to B.As for C.But for D.In addition to 37.A.chips B.pizza C.curry D.hamburgers 38.A.according to B.owing to C.in case of D.in spite of 39.A.carried B.introduced C.transported D.moved 40.A.good B.far C.important D.longPart Four Dialogue Completion

Directions : In this part, there are short incomplete dialogues between two speakers, each followed by choices marked A,B,C and D.Choose the one that most appropriately suits the conversational context and best completes the dialogue.Mark your answer on the ANSWER SHEET with a single line through the center.

41.A: I went on a date with Sammy last weekend.B: Really? _____? A: I couldn’t ask for a better night.A.Is she nice B.How was it C.Is it a beautiful night D.Did you like her

42.Student: How long can I keep the book? Librarian:___________ A.Any period longer than four weeks, you will be fined B.You can check it on the computer over there C.Four weeks, but you can renew it if you need it longer D.At most four weeks if you don’t mind

43.A: Hello, Ann, do you still remember you said you’d like to see the actor of the movie Titanic? B: __________ A: He is here at our university now.A.No, I don’t.B.Yes, why? C.Yes, so what? D.No, I don\'t like him.

44.A: Please remember me to your parents.B: ___________ A.They remember seeing you once B.Thanks a lot.I will.C.I am afraid they have poor memories D.You can contact them directly

45.Teacher: Are you following me? Student: ____________ A.I’m afraid not.B.Can you walk slowly, please? C.Let me think of it.D.Who knows?

46.Receiver: would you like to leave a meage? I’ll let Mr.Cohen know as soon as he comes back.Caller: _________ A.I hoop he will call me back B.No.I’ll call somebody else C.I don’t think it’s appropriate D.Don\'t bother.I’ll try his cell

47.David:If things go on like this, I’ll start preparing my resume again.Colleague : ____________ A: Come on.It’s not that bad. B.Yes, you are good at it.C.Really? Congratulation! D.It’s none of my busine.

48.Wife: Honey, we have only one egg and half a bottle of milk in the fridge.Oh, and we running out of toilet paper„ Husband:___________ A.Ok, ok.I’ll go to the store right now.B.Yes, honey.Let me know what I can do.C.What’s miing, dear?

D.Sure, I’ll get more next time.

49.Man: It\'s a long journey.You look tired._______? Woman: Yes.Could you take that suitcase, please? Man: Sure.A.Where’s your luggage B.How can I help you C.May I help you with anything D.Do you mind my help

50.A: Can you keep an eye on my bag: B: ___________ A: No, I’m going to the bathroom.A.Sure.What is it? B.Oh.Do you want a favor? C.Go ahead, No problem.D.Sure.Will you be long?

参考答案:

1-10:BCDCD BDDBA 11-20:DBDAC BAACC 21-30:DACBA CDBCA 31-40:BCDAC BDABC 41-50:BCBBA DAACD

推荐第7篇:GCT答案

上海千帆进管理进修学院

电话:021-63174611

网址:www.daodoc.com

逻辑答案:

01-05:BDBBB 06-10:CCADC 11-15:BCADD 16-20:DBDAA 21-25:ABACC 26-30:BDBAB 31-35:BDADA 36-40:CBAAA 41-45:ABCDA 46-50:DCCDA

推荐第8篇:GCT考试

2009年GCT考试(工程硕士)复习细则

GCT备战常识篇:

GCT是硕士学位研究生入学资格考试(英文名称为 Graduate Candidate Test,以下简称“GCT”)。国务院学位办[2005]36号《关于2005年招收在职人员攻读硕士学位工作的通知》中指出:工程硕士、农业推广硕士、兽医硕士、风景园林硕士,高等学校教师在职攻读硕士学位,中等职业学校教师等在职攻读硕士学位联考都是参加GCT考试。2008年,汉语国际教育硕士、翻译硕士加入到GCT考试。据相关人事透露,2009年将有MPA(公共管理硕士)、MPAcc等相关专业硕士学位入学资格考试加入到GCT的考试中,GCT考试将进一步发展为中国的GRE。

GCT考试内容为英语、语文、数学、逻辑四门学科,一张卷子400分,每门学科100分,其中数学25道题,英语、逻辑、语文均为50道题。考试形式全部为选择题,但要求学生在180分钟内完成175道题,题量巨大,对参加学生是一个巨大的挑战。

复习战略篇:

(1)使用权威的复习参考资料是基础。当然,有时间的话参加权威的考前辅导,可以起到事半功倍的效果。特别是逻辑科目,一是以前没学过,二是这科的解题方法和技巧确实非常重要。

(2)不能轻视任何一科,也不必过分追求任何一科,复习备考时一定要分配好自己的时间和精力。

(3)考前有针对性的训练是有必要的。必须限定时间作完题目,能否按时完成对考试的成败尤为重要。考前模拟实战训练非常重要,一定要控制在3小时内做完4科的模拟试题,从中把握考试的节奏,因为每个考生这4科的基础不一样,通过模拟训练,找出适合自己特点的答题顺序和时间分配方案,从而来确定自己现场考试的时间分配策略。

(4)同时特别要引起大家注意的是,千万不能忽略考场中的科目顺序安排,应该从实际出发,结合自己的思考及心理特点安排好所答的科目顺利,而不至于上考场发懵。先做哪一科,后做哪一科应该有个策略上的考虑。极其重要的一点是,只要把握好考试时间的分布,合理分配时间,就能最大限度地发挥自己的真实水平,就一定能有机会拿高分。

复习计划篇:

GCT考试不同于其他考试,在职人员工作繁忙,生活压力大,而且不能长时间浸泡在学习环境中。制定一个合理的学习计划,放松心情,学习是事半功倍的事,自然水到渠成。

针对以上情况,我们给在职人员几点建议:

(1)估算业余时间,根据时间长短来安排不同的科目。

(2)合理安排工作、生活。

(3)有重点的看书,找到考试方向。

(4)选择一个辅导班,会节省时间,使思路清晰。(本站将选择精品辅导班供有需要的考生参考)

(5)要有平常心,依照复习计划,坚持到最后。

阶段任务篇:

第一轮 全面备战,体会重点(6-7月)

第一遍复习前考生要浏览真题,弄清考试形式、题型设置和难易程度等内容。这有利于考生有心理准备。第一遍要把辅导教材完完全全地看通看懂,尤其数学,逻辑,第二遍再看时,就要把每章节的重点总结出来在书上标出,做笔记并复习巩固。

第二遍复习完后,考生要把所有真题的答案整理出来,如果书上没有答案,要使用网络或课外书籍。最后一遍复习时,考生要把自己复习过程中遇到的重难点都过一遍,成竹于胸。

第二轮 稳扎稳打,制成卡片(7-8月)

把数学上用到的难度大的定理、公式整理出来,附上例题,制成卡片,语文积累的知识也要总结,每日翻阅,好处是可以加深理解,减轻复习压力,还有就是把历年真题答案整理出来。

第三轮 巩固重点(8-9月)

理解记忆总结的题目,易错题,重点理论及例题等。 第四轮 考前模拟,调整心态

这段时间要从细节上考虑,查漏补缺,同时学会调节自己的心情,可以和家人一起沟通,多和大家讨论一些轻松的事情,尽量让自己开心。最重要做几套模拟试卷,衡量一下时间,做好时间精力分配。

总之,GCT考试考的是综合能力,*死记硬背去准备是根本没用的,出题很灵活,题目本身不象考研那样难,但对时间和反应速度的要求远高于考研。重点是快速反应能力,这是考试的关键所在。很多考生考试的感受是,就是觉得题目太多,做也做不完,感觉就像在抢着捡分,选一个答案就能拿两分。多数考生感觉就是来不及做,其实GCT考的是做题的速度。如果时间充足的话,即使不复习,很多考生拿个300来分都没问题。对考生来说,关键是时间不够用。因此,某种意义上比全日制研究生入学考试还残酷。所以辅导班显得越来越重要。特别是名师的讲解可以起到\"事半功倍\"的效果。

推荐第9篇:GCT逻辑

2011年GCT考试逻辑模拟试题

1 一位海关检查员认为,他在特殊工作经历中培养了一种特殊的技能,即能够准确地判定一个人是否在欺骗他。他的根据是,在海关通道执行公务时,短短的几句对话就能使他确定对方是否可疑;而在他认为可疑的人身上,无一例地都查出了违禁物品。 以下哪项如果为真,能削弱上述海关检查员的论证? Ⅰ.在他认为不可疑而未经检查的入关人员中,有人无意地携带了违禁物品。 Ⅱ.在他认为不可疑而未经检查的入关人员中,有人有意地携带了违禁物品。 Ⅲ.在他认为可疑并查出违禁物品的入关人员中,有人是无意地携带的违禁物品。 A.只有Ⅰ。 B.只有Ⅱ。 C.只有Ⅲ。 D.只有Ⅱ和Ⅲ。

2一个已经公认的结论是,北美洲人的祖先来自亚洲。至于亚洲人是如何到达北美的呢,科学家们一直假设,亚洲人是跨越在14000年以前还连结着北美和亚洲,后来沉入海底的陆地进入北美的,在艰难的迁徙途中,他们靠捕猎沿途陆地上的动物为食。最近的新发现导致了一个新的假设,亚洲人是驾船沿着上述陆地的南部海岸,沿途以鱼和海洋生物为食而进入北美的。

以下哪项如果为真,最能使人有理由在两个假设中更相信后者? A.当北美和亚洲还连在一起的时候,亚洲人主要以捕猎陆地上的动物为生。

B.上述连结北美和亚洲的陆地气候极为寒冷,植物品种和数量都极为稀少,无法维持动物的生存。

C.存在于8000年以前的亚洲和北美文化,显示出极大的类似性。

D.在欧洲,靠海洋生物为人的食物来源的海洋文化,最早发端于10000年以前。

3有着悠久历史的肯尼亚国家自然公园以野生动物在其中自由出没有著称。在这个公园中,已经有10多年没有出现灰狼了。最近,公园的董事会决定引进灰狼。董事会认为,灰狼不会对游客造成危害,因为灰狼的习性是避免与人接触的;灰狼也不会对公园中的其他野生动物造成危害,因为公园为灰狼准备了足够的家畜如山羊、兔子等作为食物。

以下各项如果为真,都能加强题干中董事会的论证,除了

A.作为灰狼食物的山羊兔子等,和野生动物一样在公园中自由出没,这增加了公园的自然气息和游客的乐趣。

B.灰狼在进入公园前将经过严格的检疫,事实证明,只有患有狂犬病的灰狼才会主动攻击人。 C.自然公园中,游客通常坐在汽车中游览,不会遭到野兽的直接攻击。

D.麋鹿是一种反应极其敏捷的野生动物。灰狼在公园中对麋鹿可能的捕食将减少其中的不良个体,从总体上有利于麋鹿的优化繁衍。

4孩子出生后的第一年在托儿所度过,会引发孩子的紧张不安。在我们的研究中,有464名12~13岁的儿童接受了特异情景测试法的测验,该项测验意在测试儿童1岁时的状况与对母亲的依附心理之间的关系。其结果:有41.5%曾在托儿所看护的儿童和25.7%曾在家看护的儿童被认为紧张不安,过于依附母亲。

以下哪项如果为真,最没有可能对上述研究的推断提出质疑? A.研究中所测验的孩子并不是从托儿所看护和在家看护两种情况下随机选取的。因此,这两组样本儿童的家庭很可能有系统的差异存在。

B.这项研究的主持者被证实曾经在自已的幼儿时期受到过长时间来自托儿所阿姨的冷漠。 C.针对孩子的母亲另一部分研究发现:由于孩子在家里表现出过度的依附心理,父母因此希望将其送入托儿所予以矫正。

2011年GCT考试逻辑模拟试题

出生后第一年在家看护的孩子多数是由祖父母或外祖父母看护的,并形成浓厚的亲情。 5在Z烟草公司的年终董事会上,董事A认为,上述统计表明,烟草业在广告上的收益正好等于其支出,因此,此类广告完全可以不做。董事B认为,由于上述10%的吸烟者所改吸的香烟品牌中几乎不包括本公司的品牌,因此,本公司的广告开支实际上是笔亏损性开支。 以下哪项,构成对董事A的结论的最有力质疑? A.董事A的结论忽视了:对广告开支的有说服力的计算方法,应该计算其占整个开支的百分比,而不应该计算其占毛收入的百分比。

B.董事A的结论忽视了:近年来各种品牌的香烟的价格有了很大的变动。

C.董事A的结论基于一个错误的假设:每个吸烟者在某个时候只喜欢一种品牌。 D.董事A的结论忽视了:世界烟草业是一个由处于竞争状态的众多经济实体组成的。 6.一家飞机发动机制造商开发出了一种新的发动机,其所具备的安全性能是早期型号的发动机所缺乏的,而早期模型仍然在生产。在这两种型号的发动机同时被销售的第一年,早期的型号的销量超过了新型号的销量;该制造商于是得出结论认为安全性并非客户的首要考虑。

下面哪个如果正确,会最严重地削弱该制造商的结论?()

A.私人飞机主和商业航空公司都从这家飞机发动机制造商那里购买发动机

B.许多客户认为早期的型号在安全性、风险方面比新型号更小,因为他们对老型号的安全性知道得更多

C.这家飞机发动机制造商的许多客户也从另一些飞机发动机制造商那里购买发动机,那些制造商在其新型号发动机中没有提供额外的安全性能保障

D.新型号的发动机可以被所有的使用旧型号发动机的飞机使用

7.来自英、法、日、德的甲、乙、丙、丁四位客人,刚好碰在一起。他们除懂本国语言外,每人还会说其他三国语言的一种。有一种语言是三个人都会说的,但没有一种语言人人都懂,现知道:

① 甲是日本人,丁不会说日语,但他俩都能自由交谈

② 四个人中,没有一个人既能用日语交谈,又能用法语交谈

③ 乙、丙、丁交谈时,找不到共同语言沟通

④ 乙不会说英语,当甲与丙交谈时,他都能做翻译

可见()。

A.甲日德、乙法德、丙英法、丁英德

B.甲日法、乙日德、丙英法、丁日英

C.甲日法、乙法德、丙英德、丁英法

D.甲日法、乙英德、丙法德、丁日德

8.真正高明的伪造家制造的钞票从不会被发现,所以一旦他的作品被认出是伪造的,则伪造者不是位高明的伪造者,真正的伪造家从不会被抓到。

下列哪种推理方式与这段话类似?()

A.田壮是一个玩魔术专家,他的魔术总能掩人耳目,从未被揭穿,所以他是一个高明的魔术师

B.王伟是一个玩魔术的人,他的魔术一般不会被揭穿,偶尔有一两次被人看穿,但这不妨碍他是一名优秀魔术师

C.岗村是一个玩魔术的人,他的魔术一般不会被人看穿,偶尔有一两次被人看穿,说明他并不是一个高明的魔术师,因为高明的魔术师不会被人看穿

D.小马的魔术很好,从不会被揭穿,所以他是一个优秀魔术师

9.生命在另外一个行星上发展,必须至少具备两个条件:(1)适宜的温度,这是与热源保持适当距离的结果。(2)至少在37亿年的时间内保持一个相对稳定的温度变化幅度。这样的条件 2

2011年GCT考试逻辑模拟试题

上述结论成立的前提是()。

A.某一个温度变化范围是生命在行星上发展的惟一必要条件

B.生命不在地球以外的地方生存

C.在其他行星上的生命形态需要的条件与地球上的生命形态相似

D.灭绝的生命形态的迹象有可能在有极端温度的行星上被发现

10.如果张未在2000年后从大学毕业,他就必须修过世界历史导论。

则这一论点是从下列哪句话中推出?()

A.在2000年前,大学学习中,世界历史导论不是必修课

B.每一个选修世界历史导论的学生都是2000年以后大学毕业的

C.没有一个2000年前毕业的大学生修过世界历史导论

D.所有2000年后毕业的大学生都必须修世界历史导论

11.一台安装了签名识别软件的电脑——这种软件仅限于那些签名在文档中的人进入计算机——不仅通过分析签名的形状,而且通过分析诸如笔尖压力和签名速度等特征来识别某人的签名。即使是最机灵的伪造者也不能复制该程序分析的所有特征。

下面哪个结论在逻辑上可以从上文得到?()

A.记录和分析某个签名需花费的时间使这种软件的日常使用变得不现实

B.安装有这种软件的计算机很快就会被大多数银行装备

C.没有人可以仅通过伪造签名的技巧而进入安装了这种软件的计算机

D.签名识别软件花费了很多年来进行发展和加以完善

12.为了增加收入,一家机场计划改变其计时停车区收取的停车费。机场会在第一个4小时或不到4小时期间收取4美元,而后每小时收取1美元;而不是在第一个2小时或不到2小时期间收取2美元,而后每小时收取1美元。

下面哪种考虑,如果正确,表明该计划可以成功地增加收入?( ) A.很少有人会在机场的计时停车区内一次停车超过2小时

B.在过去的几年内,机场运营计时停车设备的成本要高于从中获得的收入

C.把车停在机场进行短途旅行的人通常把车停在按天计费而非按时计费的停车区内

D.用来运营机场停车区的资金很大一部分被用来维护设备而不是支付收取停车费的职工工资

13.一堂考试试卷上画了五大洲的图形,每个图形都编了号,要求填出其中任意两个洲名。

甲填:3是欧洲,2是美洲

乙填:4是亚洲,2是大洋洲

丙填:1是亚洲,5是非洲

丁填:4是非洲,3是大洋洲

戊填:2是欧洲,5是美洲

结果是他们每人只对了一半。

根据以上条件下列正确的选项是()。

A.1是亚洲, 2是欧洲B.2是大洋洲, 3是非洲

C.3是欧洲, 4是非洲D.4是美洲, 5是非洲

14.王某:张健是百事可乐在中国的高级雇员之一。

李某:那怎么可能,张健只喝可口可乐。

对话中李某的陈述隐含的一个前提是()。

A.可口可乐与百事可乐同属一个总公司

B.张健在可口可乐公司并不受重用

2011年GCT考试逻辑模拟试题

张健在百事可乐兼职

D.一般的,所有的高级职员只喝本公司的产品

15.A市的政府批准了一项基金,用于建设一项发电工程,这项工程要求铺设一条管线,将山上的思乐湖的水引到附近山谷中名为田浮的小湖中。该项目所发的电本身不能平衡项目的成本,即使是进口石油——A市的电力的主要来源——的价格大幅上升时也不行。然而,这条管线是物有所值的,这是因为()。

A.石油的价格曾经历频繁的大幅上升,现在已经大幅下降,并且相当地稳定

B.该项目可重建田浮湖,该湖目前正面临干涸的危险,从而使A市面临减少一项度假收入的危险

C.A市政府目前与其进口石油的来源国的政府们都保持着良好的关系

D.将思乐湖的水引入田浮湖来进行发电的成本低于将思乐湖水引入其他山谷湖泊来发电的成本

16.罗斯镇修缮其所有的道路需要花费100万美元。但是在这些道路修缮完成之后的1年内,罗斯镇可以因此避免承担300万美元的损失,因为这个数目是现在罗斯镇每年对没有修缮的道路造成的汽车损害的赔偿额。

下面哪个,如果正确,对以上的论证提供了最强有力的支持?( )

A.与罗斯镇毗邻的社区同样对它们的道路所造成的汽车损害给予赔偿。

B.罗斯镇所有的道路都得到修缮之后,几年内不会出现道路损害汽车的情况。

C.如果罗斯镇要在1年内花费100万美元用于道路修缮,它就需要提高附加税。

D.恶劣的气候给罗斯镇的道路带来的损害程度各年之间相差很大。

17.要从代号为A、B、C、D、E、F六个侦查员中挑选若干人去破案,人选的配备要求必须注意下列各点:

① A、B两人中至少去一人② A、D不能一起

③ A、E、F三人中要派两人去④ B、C两人都去或都不去

⑤ C、D两人中去一人⑥ 若D不去、则E也不去

由此可知( )。

A.挑了A、B、F三人去B.挑了A、B、C、F四人去

C.挑了B、C、E三人去D.挑了B、C、D、E四人去

18.下面是甲、乙、丙、丁四城市某日的天气预报:已知四城市有三种天气情况,甲市和丙市的天气相同,乙市和丁市当天没有雨。

以下推断不正确的是( )。

A.甲市小雨B.乙市多云C.丙市晴D.丁市晴

19.药检局对5种抗菌素进行了药效比较,得到结果如下:甲药比乙药有效,丙药的毒副作用比丁药大,戊药的药效最差,乙药与己药的药效相同。

由此可知( )。

A.甲药与丁药的药效相同B.戊药的毒副作用最大

C.甲药是最有效的药物D.己药比甲药的药效差

20.一定的经济发展水平,只能支持一定数量和质量的人口,因而物质资料的生产和人口增长必须协调发展。人作为生产者、消费者,其数量和质量必须与生产资料的质与量、消费品的结构与数量,以及资金的数量与投资结构等相适应。

由上可以推出( )。

A.目前中国人口数量与其经济发展水平已不相适应

B.人既是生产者,又是消费者,但生产出的价值远大于消费掉的

C.提高了人的数量和质量,经济就会发展

2011年GCT考试逻辑模拟试题

当人的增长数量超过经济发展水平时,人的消费质量就会下降

21.在过去的40年里,不仅农业用杀虫剂的数量大大增加,而且农民们使用杀虫剂时的精心和熟练程度也不断增加。然而,在同一时期内,某些害虫在世界范畴内对农作物造成的损失的比例也上升了,即使在这些害虫还没有产生对现有杀虫剂的抵抗性时也是如此。

下列哪项,如果正确,最好地解释了为什么在杀虫剂使用上的提高伴随了某些害虫造成的损失更大? ()

A.在40年前通用的一些危险但却相对无效的杀虫剂已经不再在世界范围内使用了

B.由于杀虫剂对害虫的单个针对性越来越强,因此,用杀虫剂来控制某种害虫的成本在许多情况下变得比那些害虫本身造成的农作物损失的价值更大

C.由于现在的杀虫剂对特定使用条件的要求要多于40年前,所以现在的农民们对他们农田观察的仔细程度要高于40年前

D.现在有些农民们使用的某些害虫控制方法中不使用化学杀虫剂,但却和那些使用化学杀虫剂的害虫控制方法在减少害虫方面同样有效

22.在下列四个选择中,与其他三项意见差别最大的一项是()。

A.没有事物是不运动变化的B.不运动变化的事物是不存在的

C.凡事皆变D.不运动变化的事物不是不可能的

23.英、红、燕三个人讨论一数学题,当她们都把自己的解法说出来以后,英说:“我做错了。”红说:“英作对了。”燕说:“我做错了。”老师看过他们的答案并听了她们的上述意见后说:“你们三个人有一个做对了,有一个说对了”。

那么,谁做对了呢?()

A.红B.英C.燕D.不能确定

24.关于中国资本主义萌芽问题,毛泽东1939年曾写了这样一段话:“中国封建社会内的商品经济的发展,已经孕育着资本主义的萌芽,如果没有外国资本主义的影响,中国也将缓慢地发展到资本主义社会。”二三十年代的大多数中国史学家都深信:近代以前的中国已具有资本主义发展的因素。

因此()。

A.毛泽东“发明”了中国早期已有资本主义萌芽的观点

B.关于早期资本主义萌芽的观点只不过是“御用史学”的产物

C.毛泽东并没有“发明”出这种观点,而是采纳了当时大多数史学家的共同看法

D.毛泽东的观点非常正确

25.用肥皂在硬水中洗东西,肥皂将与硬水中的钙和镁发生化学反应,会产生一种不容易洗掉的沉淀物,这种沉淀不仅要浪费肥皂,而且它沉积在织物上,使织物失去光泽,产生斑点,甚至使羊毛、丝绸等织物的牢度大大降低。

这意味着()。

A.肥皂是最好的去污物B.肥皂的价格很高

C.肥皂不适合在硬水中洗东西D.肥皂在硬水里去污能力强

26.所有能干的管理人员都关心下属的福利,所有关心下属福利的管理人员在满足个人需求方面都很开明;在满足个人需求方面不开明的所有管理人员不是能干的管理人员。

由此可以推出()。

A.不能干的管理人员关心下属的福利

B.有些能干的管理人员在满足个人需要方面不开明

C.所有能干的管理人员在满足个人需要方面开明

D.不能干的管理人员在满足个人需要方面开明

27.患有行为紊乱症的动物的大脑组织中含有大量的铝元素。由于一种硅基化合物可以固定

2011年GCT考试逻辑模拟试题

上述论证基于以下哪项假设?()

A.患有行为紊乱症的动物的大脑组织中含有大量的铝元素,但其含量总保持在一定的水平上

B.行为紊乱症不会使患病动物的大脑产生比正常动物更多的铝元素

C.将这些硅基化合物引入大脑后不会有任何副作用

D.不同种类的动物需要不同量的硅基化合物来治疗

28.甲:什么是生命?乙:生命是有机体的新陈代谢。甲:什么是有机体?乙:有机体是有生命的个体。

以下哪项与上述对话最为类似?()

A.甲:什么是真理?乙:真理符合实际的认识。甲:什么是认识?乙:认识是人脑对外界的反应

B.甲:什么是逻辑学?乙:逻辑学是研究思维形式结构规律的科学。甲:什么是思维形式结构的规律?乙:思维形式结构的规律是逻辑规律

C.甲:什么是家庭?乙:家庭是以婚姻、血缘或收养关系为基础的社会群体。甲:什么是社会群体?乙:社会群体是在一定社会关系基础上建立起来的社会单位

D.甲:什么是人?乙:人是有思想的动物。甲:什么是动物?乙:动物是生物的一部分

29.教授:在长子继承权的原则下,男人的第一个妻子生下的第一个男性婴儿总是首先有继承家庭财产的权利。

学生:那不正确。休斯敦夫人是其父惟一妻子的惟一活着的孩子,她继承了他的所有遗产。

学生误解了教授的意思,他理解为()。

A.男人可以是孩子的父亲B.女儿不能算第一个出生的孩子

C.只有儿子才能继承财产D.私生子不能继承财产

30.如果某人是杀人犯,那么案发时他肯定在现场。

据此,我们可以推出()。

A.张三案发时在现场,所以张三是杀人犯

B.李四不是杀人犯,所以李四案发时不在现场

C.乙案发时不在现场,所以乙不是杀人犯

D.丙不在案发现场,但丙是杀人犯

31.最受欢迎的电视广告中有一部分是滑稽广告,但作为广告技巧来说,滑稽正是其不利之处。研究表明,虽说很多滑稽广告的观众都能很生动地回忆起这些广告,但很少有人记得推销的商品名称。因此,不管滑稽广告多么有趣,多么赏心悦目,其增加销售量的能力值得怀疑。

上文的假设条件是哪一个? ( )

A.在观众眼里,滑稽广告降低了商品信誉

B.滑稽广告虽然可看性强,但常常不如严肃的广告那样容易被人记住

C.不能使商品提高知名度的广告是不能促进销售量的增加的

D.对滑稽广告疏远的观众可能和欣赏它的观众一样多

32.由于近期的干旱和高温,导致海湾盐度增加,引起了许多鱼的死亡,虾虽然可以适应高盐度,但盐度高也给养虾场带来了不幸。

以下哪个选项为真,就能够提供解释以上现象的原因?()

A.一些鱼会游到低盐度的海域去,来逃脱死亡的厄运

B.持续的干旱会使海湾的水位下降,这已经引起了有关机构的注意

C.幼虾吃的有机物在盐度高的环境下几乎难以存活

2011年GCT考试逻辑模拟试题

水温生高会使虾更快地繁殖

33.要求60个成年人对他们的饮食记日记,日记的内容包括他们吃些什么,在什么时候以及和多少人一块吃,结果发现,在含有酒精饮料的饮食中,他们从非酒精类食物来源中摄取的热量比他们在不含酒精饮料的饮食中摄入的热量多175卡。

下面每一项,如果正确,除了哪一项之外有助于解释摄入的热量的不同?()

A.就餐者在有酒时吃饭用的时间比没酒时用的时间长

B.一天中,吃得较晚的饭倾向于比吃得较早的饭丰盛,且吃得较晚的饭包含有酒的可能性大

C.在一餐饭中,吃饭的人越多,人们吃得就越多,而且趋向于桌上有酒,吃饭的人就就多;桌上没酒,吃饭的人就少

D.在有酒的饭菜中,总的热量摄入中有相对较多的热量来自碳氢化合物,相对较少的热量来自脂肪和蛋白质

34.周庭是一名数学家,袁智是一名计算机编程专家。其实所有的编程专家都是数学家。我们知道,今天国内大多数综合型大学都在培育着计算机编程专家。

据此我们可以认定()。

A.袁智是由综合大学培养的

B.大多数计算机编程专家是由综合型大学培养的

C.周庭并不是毕业于综合大学的

D.有些数学家是计算机编程专家

35.正是因为有了第二味觉,哺乳动物才能够边吃边呼吸。很明显,边吃边呼吸对保持哺乳动物高效率的新陈代谢是必要的。

以下那种哺乳动物的发现,最能削弱以上的断言?( ) A.有高效率的新陈代谢和边吃边呼吸的能力的哺乳动物

B.有低效率的新陈代谢和边吃边呼吸的能力的哺乳动物

C.有低效率的新陈代谢但没有边吃边呼吸能力的哺乳动物

D.有高效率的新陈代谢但没有第二味觉的哺乳动物

36.甲、乙、丙和丁是同班同学。甲说:“我班同学都是团员。”乙说:“丁不是团员。”丙说:“我班有人不是团员。”丁说:“乙也不是团员。”

已知只有一个人说假话,则可推出以下判定肯定是真的一项为()。

A.说假话的是甲,乙不是团员B.说假话的是丁,乙不是团员

C.说假话的是乙,丙不是团员D.说假话的是甲,丙不是团员

37.众所周知,罗马教皇认为上帝创造世界,与此相违背的都是异端邪说。但最近教皇在一次会议上说“天主教信仰并不反对生物进化论,新的知识使人们承认,进化论不仅仅是一种假设,实际上,在继承各学科的一系列发现后,这一理论已被科学家普遍接受。” 这表明()。

A.罗马教皇已不再信仰上帝了

B.科学与神学的矛盾已不再存在

C.教皇在科学发明发达的今天,不得不承认科学的真理

D.神学已失去存在的基础和意义

38.只有在统治者不能照旧统治下去的时候,人民群众也不能照旧生活下去的时候,革命才会发生。就个人而言,选择革命或者接受革命,也往往是别的路走不通了之后,才“逼上梁山”的。

由此可见()。

A.任何一次革命,都不是人为制造的结果

B.革命不需要领导力量也会成功

2011年GCT考试逻辑模拟试题

革命不需要动员,群众会自发参与的

D.所有在个人生活中遇到挫折的人都会参加革命和支持革命

39.售价2元一斤的洗洁精分为两种:一种加有除臭剂,另一种没有除臭剂。尽管两种洗洁精的效果相同,但没加除臭剂的洗洁精在持续时间方面明显不如有除臭剂的洗洁精。

因此后者()。

A.味道要好闻些B.具有添加剂

C.从长远来看更便宜D.比其他公司的产品效果好

40.目前,生物加工食物体系要成为现实,还有几件事要做。先得能在培养基地培养出蔬菜和水果的组织,还得能扩大产量以应市场之需。而要大量经济地生产,还得研制出廉价原料,利用其中的基本营养成分进行批量生产。由此可以推出()。

A.研制出廉价原料是生物加工食物体系的关键

B.目前已经能在培养基地培养出蔬菜和水果的组织

C.只要有廉价的原料就能扩大产量

D.目前的原料太昂贵

在一个标有R,S,W,X,Y,Z6个国家的地图上,相邻的国家不能是相同的颜色。相邻国家如下描述:

(1)R,S,X,Y与W相邻 (2)X与Y相邻

(3)R和S与Z 相邻

41 下列哪一对国家颜色必须不同?

A. R和X

B. S和X

C.S和Z

D.X和Z 42 假如X和Z 颜色相同,那么下列哪一项必然正确? A.R和Y颜色相同

B.S和X颜色相同

C.X和Y颜色相同

D.W和其他任何国家的颜色都不相同 43 下列哪一对国家颜色可以相同?

A. R和S

B S和W

C.W和X

D.W和Y 44. 下列哪一个国家的颜色可以与W的颜色相同?

A .R

B.S

C.,X

D, Z 45.最少可以用几种颜色来标示这些国家?

A. 2

B.3

C. 4

D.5 一块平坦的野地有4个不同的隐蔽处即F,G,W,X,他们分别由4条道路联结,这4条道路的名字分别是Q,R,S,T,并且长度相同,以下列方式联结: Q仅仅联结F和W R仅仅联结G和W S仅仅联结F和G T仅仅联结G和X 46 下列哪项是一名徒步者从F 出发仅仅使用小路,并且使用小路不超过一次必须到达其他隐藏处的顺序?

A.GWX

B.WGX C.WXG

D.XGW 47.假如一个徒步者从X到达F ,可以选择的最短长度的到达方法有几种?

A. 1

B. 2

C.3

D.4 48.假如一个徒步者限定自己只走小路,下列除了哪一个是走小路,且走完小路全长的可能序列?

2011年GCT考试逻辑模拟试题

A.QSRTS B.RQSRQ C.STTRQ

D.TSQRT 49 假如一个徒步者途径每条小路的全长一次,下列哪一个列出了所有的隐藏处,并且仅有那些隐藏处,徒步者必须经过两次?

A.G

B. F和G

C. G和W

D. G和X 50.假如通过走离开小路的近道,一位徒步者可以从W到X走一条比仅仅走小路从W到X的最短距离还要近的距离,下列哪一个一定正确?

A.从F 到X的最短距离要比从W到X的最短距离要短。 B. F和X之间的最短序列是F和X之间的最短距离。 C.通往R和T的不是一条直线。 D.通往S和T的不是一条直线

1-5 DBADD

6-10 BACCD

11-15 CACDB 16-20 BBCDD

21-25 BDCCC

26-30 CBBCC

31-35 CCDDD

36-40 ACACD 41-45 CDADB

46-50 BAAAC

51.在计算机语言中有一种逻辑运算,如果两个数同一位上都是0时,其和为0,一个为0,一个为1时或两个都是1时,其和为1。

那么(B)。

A.如果和为1,则两数必然都是1 B.如果和为0,则两数必然都为0

C.如果和为0,则两数中可能有一个为1

D.如果和为1,则两数中至少有一个为0

52.派出所民警讯问公共汽车上的一桩盗窃案的嫌疑人甲、乙、丙、丁的笔录如下:

甲说:“反正不是我干的” 乙说:“是丁干的。” 丙说:“是乙干的。” 丁说:“乙是诬陷。”

他们当中只有三人说真话,扒手只有一个,是(B)。

A.甲 B.乙 C.丙 D.丁

53.若风大,就放飞风筝;若气温高,就不放飞风筝;若天气不晴朗,就不放飞风筝。

假如以上说法正确,若放飞风筝,则以下哪些说法是正确的?(B) ① 风大② 天气晴朗③ 气温高

A.① B.② C.③ D.①和②

54.1980年美国人口普查结果显示,婚姻状况中分居(包括法律上的分居和两地分居)的女性比男性多100万,以下哪项有助于解释此结果?(D) ① 在美国婚龄女性比婚龄男性多

② 人口普查漏掉的分居男性多于分居女性

③ 有更多的分居男性出国居住

A.只有① B.只有② C.②和③ D.①和③

55.小董并非既懂英文又懂法语。

如果上述断定为真,那么下列哪项断定必定为真?(D) A.小董懂英文但不懂法语

B.小董并懂法语但不懂英文

C.小董既不懂英文又不懂法语

D.如果小董懂英文,小董一定不懂法语

推荐第10篇:GCT考试英语完形填空专项练习题(C)

2013年GCT考试英语完形填空专项练习题(C) 来源:中国在职教育网 www.daodoc.com 第一篇:

Mr.Green was ill and went to the hospital.A doctor __1__ and said, “Well, Mr.Green, you are going to __2__ some injections, and you’ll feel much better.A nurse will come __3__ give you the first one this evening, and then you’ll __4__ get another one tomorrow evening.” __5__ a young nurse came to Mr.Green’s bed and said to him, “I am going to give you your __6__ injection now, Mr.Green.Where do you want it?”The old man was __7__.He looked at the nurse for a __8__, then he said, “__9__ has ever let me choose that before.Are you really going to let me choose now?”

“Yes, Mr.Green,” the nurse answered.She was in a hurry.“Where do you want it?”

“Well, then,” the old man answered __10__ “I want it in your left arm, please.”

1.A.looked for him B.looked him over

C.looked after him D.looked him up

2.A.get B.give C.make D.hold

3.A.so B.but C.or D.and

4.A.must B.can C.had better D.have to

5.A.In the morning B.In the afternoon

C.In the end D.In the evening

6.A.first B.one C.two D.second

7.A.confident B.surprised C.full D.hungry

8.A.hour B.minutes C.year D.moment

9.A.Somebody B.Anybody C.Nobody D.people 10.A.with a smile B.in time C.in surprise D.with tears in his eyes

1.B。look for sb/sth 意为“寻找……”;look after sb 意为“照料……”;look up sb意为“看望……”;而look over sb意为“检查某人”,最贴近文意,为正确选项。

2.A。医生要对格林先生进行注射,格林先生是动作的接受者,故应选get。

3.D。空白部分前面I come 和后面的give形成承接关系,所以应用 and连接。

4.D。must 不可以用于将来时,根据文章意思,应选have to。

5.D。与上文this evening相对应,In the evening应为正确选项。

6.A。one 填入空白部分显得画蛇添足,根据上下文这是第一次注射,应用first。

7.B。老人对护士的提问应感到surprised, 因为下文提到从来没人问过他这样的问题。

8.D。对护士的提问, 老人思考了一会儿, 故应选moment。

9.C。老人感到奇怪, 是因为没有人问过这样的问题, 故应选nobody。

10.A。老人想捉弄一下这个小护士。按常理, 应是带着微笑取笑她, 故应选with a smile。 第二篇:

Mr.and Mrs.Harris had always spent their summer holidays in a small hotel at the seaside near their hometown.One year, however(然而) , Mr.Harris made a lot of 1 in his busine, 2 they decided to go to a foreign country and stay at a really good 3 .更多资源请参见:中国在职研究生网 www.daodoc.com 竟成教育考前辅导:www.daodoc.com

They flew to Rome, and 4 at a 5-star hotel late in the evening.They thought they would have to go to bed hungry, because in that 5 hotel where they had been used to stay in the past, no meals were served (供应) 6 seven in the evening.They were 7 to be told that the hotel served dinner until ten.

“Then what are the times 8 meals?” asked Mrs.Harris.

“Well, madam, we serve breakfast from seven to eleven, lunch from twelve to three, 9 from four to five, and dinner from six to ten.”

“But that hardly 10 any time for us to see the city!” said Mrs.Harris.

1.A.mistakes B.time C.friends D.money

2.A.but B.so C.though D.yet

3.A.hotel B.place C.city D.restaurant

4.A.stayed B.got C.arrived D.reached

5.A.small B.big C.foreign D.good

6.A.on B.after C.during D.until

7.A.tired B.interested C.surprised D.worried

8.A.with B.on C.at D.of

9.A.drink B.tea C.beer D.food

10.A.takes B.does C.has D.leaves 参考答案:

1.D。根据下文,哈里斯夫妇出国旅游并住进高级宾馆,说明他们赚了很多钱。故选money。

2.B。made a lot of money与下文go to a foreign country构成因果关系, 故选so。

3.A。与下文他们住进a 5-star hotel相对应,这里应选择hotel。

4.C。表示到达目的地,reach可直接接宾语, get后应加介词to, arrive为不接物动词,后应加介词in或at才能接宾语,所以arrive为正确选项。

5.A。根据文章的第一句,以前他都是住的small hotel,这里应选small。

6.B。on和during都不能与seven连用。他们估计要挨饿,说明在以前住的宾馆里7点以后不可能有饭菜供应。故应选择after。

7.C。有人告知饭菜供应会持续到十点时,他们理应感到surprised。

8.D。of常用来表示所有关系,the times of the meals表示“每顿饭的供应时间”。

9.B。根据西方人的生活习性,宾馆一般不会在一段时间只供应啤酒,下午喝茶比较合乎常理。故选tea。

10.D。哈里斯以为从早到晚都得在吃饭或喝茶,几乎没有剩余时间观光旅游了,leave表示“剩下,遗留”,为正确选项。 第三篇:

Long ago there was a poor farmer called Fred.Fred and his wife, Doris lived 1 together in their small old house.One winter night, the Luck Fairy (仙女) visited them .

“Fred, you’re a 2 farmer.I’d like to give you a wish,” said the Luck Fairy.更多资源请参见:中国在职研究生网 www.daodoc.com 竟成教育考前辅导:www.daodoc.com

“A wish?” Said Fred.

Fred and Doris smiled at each other.Then Fred said, “ Thank you , Luck Fairy.We’re very 3 and happy.”

“ 4 we’re old, we still work in the field every day,” said Doris.

“You wok very hard but you 5 very little money.Would you like some gold coins ” asked the Luck Fairy.

“Oh no , my dear Luck Fairy.We’re poor.But we have 6 food to eat.” Replied Fred.

“You can use the gold coin to buy some clothes.The winter here is very cold,” said Luck Fairy.

“Though we haven’t got 7 clothes, we’ve got enough,” said Doris.

“Well, what about a nice new house?” Asked Luck Fairy.

“Thank you, but I 8 my small old house very much.I’ve lived here since I was born.I don’t 9 a new house,” said Fred.

“You’re quite different from other people.I like you very much,” said the Luck Fairy.“I wish you happine and Luck forever.” Then the Luck Fairy 10 and never came back.

1.A.sadly B.happily C.worried D.anxiously

2.A.bad B.lazy C.good D.unhelpful

3.A.healthy B.careful C.difficult D.important

4.A.If B.But C.Because D.Though

5.A.cost B.lose C.make D.borrow

6.A.no B.little C.enough D.expensive

7.A.old B.many C.bad D.clean

8.A.hate B.love C.need D.dislike

9.A.need B.see C.buy D.build

10.A.smiled B.nodded C.laughed D.disappeared 答案解析:

1.B。根据下文我们知道,这对夫妇生活得很愉快。

2.C。正因为Fred是一个好农夫,仙女才要奖励他。

3.A。比较这四个词的意思不难发现与happy 并列的是healthy。

4.D。根据still可知选though。虽然他们年纪大了,但仍然能够在田里干活。

5.C。make money意思是“赚钱”。

6.C。根据文意,他们对一切都感到知足,包括食物他们也觉得足够吃了。

7.B。他们没有许多衣服,但对他们来说却已经够穿了。

8.B。从后面的句子可知,他是喜欢那个房子的。

9.A。根据上文,他们喜欢自己的小屋,所以不需要新的。

10.D。根据never come back可知仙女消失了。 第四篇:

Once, a king showed two men a large basket in the garden.He told them to fill it with 更多资源请参见:中国在职研究生网 www.daodoc.com 竟成教育考前辅导:www.daodoc.com

water from a well.After they __1__ their work, he left them, saying, “When the sun is down, I will come and see your work.”

At last one of them said, “What′s the use of doing this foolish work? We can __2__ fill the basket.” __3__ man answered, “That is none of your busine.” The first man said.“You may do as you like, but I am not going to work at __4__ so foolish.” He __5__ his bucket and went away.The other man said no word, and kept on carrying __6__.At last the well was almost __7__.

As he poured the last bucket of water into the basket, he saw a bright thing in it.He picked it up.It was a beautiful gold ring.Just then the king came.__8__ he saw the ring, he knew that he had found the kind of man he wanted.He told him to keep the ring for himself.“You __9__ so well in this little thing,” he said, “ __10__ now I know I can believe you with many things.”

1.A.finished B.did C.began D.had

2.A.ever B.never C.easily D.no

3.A.The other B.Another C.One D.A second

4.A.anything B.something C.nothing D.everything

5.A.picked up B.put away C.took away D.threw away

6.A.water B.basket C.well D.work

7.A.full B.empty C.filled D.clean

8.A.While B.As soon as C.Before D.Since

9.A.have done B.will do C.do D.are doing

10.A.what B.why C.when D.that

答案简析

1.C。这里did和finished都表示完成了这项工作,而给篮子装满水是不可能的,国王应在两人开始打水后不久离开,所以应选began。

2.B。往篮子里盛满水是永远不可能的,故选择never。

3.A。两者中的另一个用the other加名词来表示。

4.C。the first man想离开,因为他觉得自己干的是无用功,故选择nothing意为“从事某项工作”。

5.D。pick up意为“捡起”,pick away意为“放好”,take away意为“取走”,而throw down意为“扔掉、丢弃”,比较贴近文意,为正确选项。

6.A。根据文意,另一个人一直在打水,故选water。

7.B。不停地打水必然会导致井空,故选empty。

8.B。while引导从句时,从句应用延续性动词,before和since不符合文意,as soon as…表示“一……就”为正确选项。

9.A。国王是因为这个诚实的人所做过的事情而表扬他,所以应用完成时态have done。

10.D。国王讲的最后一句话是含有 “so……that”结构的复合句,意为“如此……以致”故选that。

更多资源请参见:中国在职研究生网 www.daodoc.com 竟成教育考前辅导:www.daodoc.com

第11篇:GCT考试英语强化训练模拟试题(三)

2009年GCT考试英语强化训练模拟试题

(三)

Part Three Cloze

Directions:

There are ten blanks in the following paage .For each numbered blank , there are four choices marked A,B,C and D.Choose the best one and mark your answer on the ANSWER SHEET with a single through the center.

“Without time to relax and have fun, kids can suffer stre just like adults,” warn experts---who say as many as one in four youngsters have symptom of burnout.More and more parents are pushing their kids to be busy in structured activities 31 ______ .

Many of these activities for children are not for recreational 32 ______ involve competition.The kids 33 ______ to “win”, not just participate, and this can cause34 ______ .Today’s parents have the 35 ______ that children don’t pursue a lot in outside activities will be36 ______ .Parents are in panic because they know it’s a 37 ______ world out there.They are running scared to be sure their kids can go into marketplace and compete as adults, but38 ______ some cases they are miing the big picture.Kids who are unhappy and depreed grow up to be unhappy, despre adults who don’t do well in their jobs or 39 ______ life.And these kids will not know as adults how to relax.Everyone needs time just to relax and recharge.When you are not streed, you can be 40 ______ productive.

That’s why it’s important to help your child find a balance.

31.A.at no time B.at times C.all the time D.at one time

32.A and B.but C so D as

33.A push B are pushed C have pushed D are being pushed

34.A failure B threat C stre D diligence

35.A mood B attitude C style D idea

36.A left alone B left behind C left out D left over

37.A pleasant B tough C colorful D adventurous

38.A in B at C under D on

39.A personal B public C outside D social

40.A very B most C more D le

Part Four Dialogue Completion

Directions:

In this part, there are ten short incomplete dialogues between two speakers, each followed by four choices marked A,B,C and D.Choose the one that most appropriately suits the conversational context and best completes the dialogue.Mark your answer on theANSWER SHEET with a single line through the center.

41.Speaker A: Sam, I’m calling to say goodbye to you, as I’m leaving this afternoon.

Speaker B: _____

A.Look after yourself, and thank you for your calling.

B.Pay attention to your schedule.Don’t be late for the train.

C.Thank you and don’t forget to keep in touch with me.

D.Take care and I wish you a pleasant journey.

42.Speaker A: Congratulations! I hope you’ll be very happy.

Speaker B: _____

A.Oh, that is easy.

B.We will.

C.Why not.

D.Thanks, I am sure we will.

43.Speaker A: I need some aspirin, please, and I\'d also like to get this prescriptionfilled.

Speaker B: _____

A.Fine.Here\' s your aspirin.Could you wait a moment and I can have the prescription for you.

B.Well.I don’t think you need any aspirin.I don\' t think you need any more prescription.

C.Aspirin? Here you are.Here is your prescription, too, if you like any.

D.Don\' t you need any aspirin?Here you are.As for the prescription, I haveto think it over.

44.Speaker A: If I were youI\' dtake the bus to work.Driving in that rush-hour traffic is terrible.

Speaker B: _____

A.Oh, no.Driving in that rush-hour traffic is terrible though the bus is crowded.

B.Well.I do agree with you even ifthebus is often late.

C.But by the time the bus gets to my stop, there aren’t any seals left.

D.No.Do you mind if I contradict you.I like driving in that rush-hour.

45.Speaker A: Shells is an American, but she\' s lived in the Far East for most of her life.

Speaker B: _____

A.Yes.I know she is an American.But what about an American?

B.Yes.But an American is merely anAmerican.Do American speak English?

C.Yes.But don\' t you think if she can speak American?

D.Yes.She speaks Japanese and Chinese as well as she speaks English.

46.SpeakerA: Could I borrow your bikethis weekend?

SpeakerB: Sure.______

A.You can useit anytime you want.

B.But you have to pay me something back.

C.What do you need it for?

D.Where are you going?

47.Speaker A: Could I see a pair of sandals like the brown ones in the window? I need a size six-and-a-half.

Speaker B: _____

A.I\' m sorry but that style doesn’t come in half size.Ican show you a seven.

B.That pair of sandals? Why do you like that pair of sandals?

C.Do you like the pair of sandals like the brown ones inthewindow? Fine, thank you.

D.Isee what you mean.But I am afraid that that pair will be tooexpensiveto you.

48.Speaker A:After the terrible downpour last night, It’s lovely today, isn’t it?

Speaker B: _____

A.Yes, isn\'t it?

B.Yes, it is.

C.That\'s not too bad.

D.It\'s hard to say.

49.SpeakerA: ______

SpeakerB:What\'s the problem?

SpeakerA: Iswitched on the power five minutes ago, and now it doesn’t show up.

A.Are youthe owner of the computer here?

B.Do you know whom this computer belongs to?

C.What can I do for you?

D.Could you help me?

50.Speaker A:After the terrible downpour last nigh, It’s lovely today, isn’t it?

Speaker B: _____

A.Yes, isn\'t it?

B.Yes, it is.

C.That\'s not too bad.

D.It\'s hard to say.

第12篇:GCT考试英语强化训练模拟试题(二)

2009年GCT考试英语强化训练模拟试题

(二)

Questions 21-25 are based on the following paage:

In the Arctic Circle, it is not that Eskimos lack ability or industry, but the surroundings restrict constructive effort to the barest neceities of existence.This retards progre to higher develpment.

Agriculture is impoible all along the thousands of miles of the north shore.The only wood is such as drifts in.other than this driftwood, the only available building materials are snow, ice, stone, and bones of animals.All of these have been used for habitations and storage, places, differing in various tribes according to the requirements and skill of the workers.The lack of neceary timbers to build walls and span wide spaces is probably one reason why they construct their houses at least partly beneath the surface of the ground.This device also makers the houses more impervious (不能渗透的) to the cold.

Most of us are inclined to think that the Eskimo lives always in an igloo or snow house.This is not entirely true.After the long cold winter, the family is very likely to move, when the weather permits, into a tent sealskin.The actual construction of such a tent is similar to that used by other, more southerly tribes and will be described later.

The snow house, however, is an interesting and unique habitation.Our summer campers will not build, with snow, but the delicate art is worth recording and some of our winter camps mountains might try to make snow houses.

21.Eskimos’ efforts to build houses ___________.

A.result in various building

B.are limited by a hostile environment

C.are restricted by their ability

D.retard progre to higher develpmen

22.Which of the following about the construction of houses is true?

A.Building materials differ from tribe to tribe.

B.Building materials are the same for all the houses.

C.Building materials are selected according to weather.

D.Building materials are decided by skilled workers.

23.Why do Eskimos build their houses partly under the ground?

A.They like to live under the ground.

B.They are short of eential materials for walls and roofs.

C.They want their houses le affected by the cold.

D.Both B and C.

24.In the long winter Eskimos commonly live in _________.

A.a snow house

B.a stone house

C.a storage place

D.a tent sealskin

25.What does the author think of snow houses? A.Interesting B.Artistic C.Unique D.All of the above Questions 26-30 are based on the following paage: Atlanta: Mostly fair.88-70 0 F.Minneapolis: Mostly cloudy.68-50 0 F. Boston: Partly cloudy.78-61 0 F.New Orleans: Mostly fair.92-73 0 F. Chicago: Thunderstorms likely.82-67 0 F.Philadelphia: Hazy and warm.90-68 0 F. Cleveland: Mostly cloudy.84-68 0 F.Phoenix: Sunny and warm.99-66 0 F. Dallas: Thunderstorms likely.91-75 0 F.Pittsburgh: Partly cloudy.81-64 0 F. Denver: Rain likely.53-43 0 F.St.Louis: Thunderstorms likely.86-70 0 F. Houston: Partly cloudy.90-78 0 F.San Francisco: Mostly fair.73-60 0 F. Kansas City: Thunderstorms likely.73-60 0 F.Seattle: Mostly fair.74-50 0 F. Las Vegas : Sunny and warm.93-56 0 F.Toronto: Rain likely.83-68 0 F. Los Angeles: Mostly sunny.88-60 0 F.Washington: Partly cloudy.88-72 0 F. Miami: Partly cloudy.88-79 0 F. 26.According to the forecast, A.Kansas City will be warmer than Toronto B.Temperatures will be the lowest in Seattle and Minneapolis. C.The weather will be fine in most ofthe listed cities. D.More than half of the listcities are cloudy or rain likely 27.Among the following four cities.The difference between the afternoon high and the evening low is smallest in

A.Kansas City

B.Miami

C.Atlanta

D.Seattle

28.Which of the following cities is closest to Miami in weather conditions?

A.Boston.

B.Atlanta.

C.Washington.

D.Houston.

29.The difference between the afternoon high and the evening low is greatest in ______ .

A.Seattle

B.Los Angeles

C.Las Vegas

D.Phoenix

30.The differences between the afternoon high and the evening low are the same in ______ 。

A.Chicago.Dallas, Kansas City, and St, Louis

B. Atlanta, San Francisco, New Orleans ,and Seattle.

C. Cleveland, Dallas, St, Louis, and Washington.

D.Pittsburgh, Miami, Houston, and Boston

第13篇:GCT考试(英语)真题及答案

第四部分 外语运用能力测试(英语) (50题,每题2分,满分100分) Part One Vocabulary and Structure Directions: There are ten incomplete sentences in this part.For each sentence there are four choices marked A, B, C and D.Choose the one that best completes the sentence.Mark your answer on the ANSWER SHEET with a single line through the center.1.He added that the state government has made _________ arrangements for the conference. A.accurate B.absolute C.adequate D.active 2.This video may be freely reproduced __________ commercial promotion or sale. A.as for B.except for C.thanks to D.up to 3.You ___________ engage in serious debate or discuion unle you are willing to endure attacks. A.have better not B.had better not C.have better not to D.had better not to 4.Coffee has been a favorite drink for centuries, _________ the time when we were drinking it strong and black, without sugar. A.during B.for C.before D.since 5.By 2050 the world will have about 2 billion people aged over 60, three times _________ today. A.as much as B.as that of C.as many as D.as those of 6.Saffron returned to London to __________ her acting career after four years of modeling. A.follow B.chase C.seek D.pursue 7.He has fancy dreams about his life, and nothing ever quite ____________ his expectations. A.matches B.makes C.reaches D.realizes 8.___________ my neighbor\'s kid with his coming exam, I spend an hour working with him every day. A.To help B.Helping C.Helped D.Having helped 9.When I worked as a bank clerk, I had the opportunity to meet a rich ___________ of people: students, soldiers and factory workers. A.diversity B.kind C.range D.variety 10.Cuts in funding have meant that equipment has been kept in service long after it _____________ replaced. A.should have been B.would have been C.could have been D.might have been Part Two Reading Comprehension Directions: In this part there are three paages and one chart, each followed by five questions or unfinished statements.For each of them, there are four choices marked A, B, C and D.Choose the best one and mark your answer on the ANSWER SHEET with a single line through the center.Questions 11-15 are based on the following paage: Happy hours are not necearily happy, nor do they last for an hour, but they have become a part of the ritual of the office worker and busineman. On weekdays in pubs and bars throughout America, there is the late afternoon happy hour.The time may vary from place to place, but usually it is held from four to seven.After the workday is finished, office workers in large cities and small towns take a relaxing pause and do not go directly home.They head off instead for the nearest bar or pub to be with friends, co-workers and colleagues.Within minutes the pub is filled to capacity with businemen and secretaries, office clerks and stock executives.They gather around the bar like birds around a fountain or forest animals around a watering hole and chat about the trifles of office life or matters more personal.This is their desert garden, the place to relieve the day\'s stre at the office. At these happy hours, social binding occurs between people who share the same workplace or similar profeions.They may chat about each other or talk about a planned project that has yet to meet a deadline.In this sense, these places become extensions of the workplace and constitute a good portion of one\'s social life.11.For office workers and businemen the happy hour is their ____________. A.profeional requirement B.regular practice C.refreshing break D.unpaid work 12.Happy hours are held because office workers need to ___________. A.have a good rest after work B.stay away from household work C.make new friends D.celebrate their achievements 13.The phrase \"filled to capacity\" in Paragraph 2 means the pub is _____________. A.too crowded B.rather entertaining C.completely full D.very noisy 14.Happy hours contribute to office workers\' __________. A.cooperation in society B.promotion in their company C.connection in society D.loyalty to their company 15.Which of the following statements is NOT true? A.The happy hour is a social gathering in America. B.People avoid talking about work at happy hours. C.Happy hours are held on weekdays only. D.People exchange work experiences at happy hours.Questions 16-20 are based on the following paage: Lazy? Shy? Live in a cave? Those might not be positive attributes for the average human, but they sure are good for animals trying to survive in a changing environment.According to a new study, beasts that hibernate (冬眠) or crawl into holes are le likely to be listed as endangered than those that don\'t. Following up a previous study on extinct animals, which showed that species exhibiting \"sleep or hide\" (SLOH) behaviors did better than others, the researchers wanted to see if the same was true of modern creatures like moles and bears.To find out if our more timid animals have a leg up in the survival game, researchers made a master list of 443 sleep-or-hide mammals. With their list in hand, the team compared their 443 to the \"red list\" of endangered species published by the International Union for Conservation of Nature.As suspected, a sleepy or hiding animal was le likely to be on the red list than a regular animal, and a red-list animal was also le likely to be a SLOH-er. This makes a lot of sense, as animals that hide away in a cave or a tree hole are protected by their physical shelters from a variable environment outside, while hibernators enjoy a flexible metabolism (新陈代谢) that can help them adapt to a changing climate.16.On the list of extinct animals studied, there were _____________. A.fewer SLOH-ers than regular animals B.more SLOH-ers than expected C.as many SLOH-ers as regular animals D.hardly any SLOH-ers 17.The phrase \"a leg up\" in Paragraph 2 probably means __________. A.an instinct B.an advantage C.a fight D.a chance 18.The study of modem creatures ___________. A.is unrelated to the study of extinct animals B.finds evidence miing in the study of extinct animals C.has findings similar to those of the study of extinct animals D.reveals a different pattern from the study of extinct animals 19.According to the paage, red-list animals are more likely to ____________. A.be lazy B.be timid C.live long D.sleep le 20.In the last paragraph the author ____________. A.compares the behaviors of sleepers and hiders B.offers an explanation for the survival of sleepers and hiders C.analyzes how a changing environment affects SLOH-ers D.emphasizes what can be learned from SLOH-ers Questions 21-25 are based on the following paage: In computing, pawords are commonly used to limit acce to official users.Yet the widespread use of pawords has serious drawbacks.Office workers now have to remember an average of twelve system pawords.In theory they should use different pawords for each site, but in reality these would be impoible to remember, so many people use the same paword for all. An additional problem is that the majority use simple words such as \"hello\", or names of family members, instead of more secure combinations of numbers and letters, such as 6ANV76Y.This permits computer hackers to download dictionaries and quickly find the word that allows them acce. When system users forget their pawords there is extra expense in supplying new ones, while if people are forced to change pawords frequently they often write them down, making systems even le secure.Therefore, it is clear that the idea of pawords, which have been used as security devices for thousands of years, may need rethinking. One poible alternative has been developed by the American firm Real User, and is called \"pafaces\".In order to acce the system a worker has to select a series of photographs of faces from a randomly (随机地) generated sequence.If the pictures are selected in the correct order, acce is granted.This concept depends on the human ability to recognize and remember a huge number of different faces, and the advantage is that such a sequence cannot be told to anyone or written down, so is more secure.It is claimed that the picture sequence, which used photographs of university students, is easier to remember than pawords, and it has now been adopted for the United States Senate.21.What is the disadvantage of pawords as mentioned in Paragraph 1 ? A.They do not ensure security. B.They are difficult to remember. C.They have to be changed frequently. D.They limit computer acceibility.22.One can make a paword safer by _____________. A.inserting pictures between numbers B.avoiding the use of letters altogether C.setting up a firewall against computer hackers D.using complicated combinations of numbers and letters 23.\"Pafaces\" is a method to get acce to a system through ___________. A.remembering a large number of faces B.selecting photographs of faces one likes C.recognizing a sequence of face pictures D.showing one\'s face in front of the computer 24.One advantage of \"Pafaces\" over a paword is that ____________. A.it is easier to remember B.it is more complicated C.it takes le time to log in D.it allows one to write le 25.What does the author think of the paword? A.R is an old system that needs improvement. B.It provides as much security as before. C.R should be abandoned by computer users. D.It has developed to an advanced stage.Questions 26-30 are based on the following chart: FedEx Service Restrictions U.S.EXPRESS FREIGHT INTERNATIONAL EXPRESS FREIGHT INTERNATIONAL AIR CARGO 1 or 2 Day Freight 3 Day Freight International Priority Freight or Economy Freight International Premium or Expre Freight International Airport to Airport Minimum weight per piece or shipment 68kg 68kg 68kg No minimum restrictions No minimum restrictions Maximum weight per piece 997kg 997kg 997kg 997kg 997kg Maximum length Plus girth per piece 762cm 762cm 762cm 762cm 762cm Maximum length per piece 302cm 302cm 302cm 302cm 302cm Maximum height per piece 178cm 178cm 178cm 178cm 178cm

26.Which of the following might be a proper title for the chart? A.FedEx Freight Measurements and Methods B.FedEx Shipment Regulations in US and Other Countries C.FedEx International Freight Customer Service Guide D.FedEx Expre Freight and Air Cargo Service Restrictions 27.What\'s the minimum weight a shipment must reach in order to be transported by air? A.No restrictions.B.68kg. C.122kg.D.997kg.28.If you need to ship something 300cm long within US, which service can you choose? A.Shipment in le than 1 day. B.Shipment in 1 or 2 days. C.Shipment in 3 days. D.Shipment in more than 3 days.29.What does \"girth\" poibly mean? A.Measurement around an object. B.Measurement of object weight. C.Formula to calculate object width. D.Formula to calculate object length.30.FedEx services have different restrictions on the goods\' ____________. A.maximum weight per piece B.maximum length plus girth per piece C.maximum length per piece D.maximum height per piece Part Three Cloze Directions: There are ten blanks in the following paage.For each numbered blank, there are four choices marked A, B, C and D.Choose the best one and mark your answer on the ANSWER SHEET with a single line through the center. Fueled by weather, wind, and dry undergrowth, uncontrolled wildfires can burn acres of land—and consume everything in their way—in mere minutes. 31 , more than 100,000 wildfires clear 4 million to 5 million acres of land in the U.S.every year.A wildfire moves at speeds of up to 23 kilometers an hour, consuming everything—trees, bushes, homes, even humans—in its 32. There are three conditions that need to be 33 in order for a wildfire to burn: fuel, oxygen, and a heat source.Fuel is any material 34 a fire that will burn quickly and easily, including trees, graes, bushes, even homes.Air supplies the oxygen a fire 35 to burn.Heat sources help spark the wildfire and bring fuel to 36 hot enough to start burning.Lightning, burning campfires or cigarettes, hot winds, and even the sun can all provide 37 heat to spark a wildfire. 38 often harmful and destructive to humans, naturally occurring wildfires play a positive role in nature.They 39 nutrients to the soil by burning dead or decaying matter.They remove diseased plants and harmful insects from a forest ecosystem (生态系统).And by burning 40 thick trees and bushes, wildfires allow sunlight to reach the forest floor, enabling a new generation of young plants to grow.31.A.After all B.Above all C.In sum D.On average 32.A.route B.track C.path D.trace 33.A.stable B.present C.fixed D.favorable 34.A.surrounding B.keeping C.causing D.making 35.A.acquires B.needs C.captures D.meets 36.A.materials B.places C.temperatures D.conditions 37.A.additional B.exceive C.plentiful D.sufficient 38.A.Although B.As C.If D.Whereas 39.A.drive B.reduce C.return D.aign 40.A.over B.through C.below D.beyond Part Four Dialogue Completion Directions: In this part, there are ten short incomplete dialogues between two speakers, each followed by four choices marked A, B, C and D.Choose the one that most appropriately suits the conversational context and best completes the dialogue.Mark your answer on the ANSWER SHEET with a single line through the center.41.Speaker A: Hi.My name is Mark.I\'m from Houston, Texas. Speaker B: I\'m Bill.Glad to meet you.What year are you? Speaker A: ___________. A.I was born in 1990 B.I\'ve been here for years C.I\'m 19 years old D.I\'m a first-year student 42.Speaker A: I\'m getting pretty bored.We should do something despite the rain. Speaker B: ____________.What do you have in mind? A.I back you up.B.Who cares? C.I\'m with you.D.I like the rain.43.Man: We had a trip to South Africa this summer. Woman: ___________. Man: Yes, we did.In fact, we even encountered a lion. A.Didn\'t you? B.How did it go? C.I bet you had a great time.D.I gue you did.44.Man: Do you know Jason\'s phone number? Woman: ____________. Man: OK.I might as well look it up in the phone book. A.Just a second.B.Not that I know of. C.I can\'t think of it now.D.Why ask? 45.Interviewer: Let me see if I understood you.You mean that you can work extra hours if needed, right? Interviewee: ______________. A.Yes.No matter what you say.B.Yes.Thank you for your clarification. C.Yes.You sure understand me.D.Yes.Absolutely.46.Speaker A: Thanks to John, we\'ve lost our most important client. Speaker B: I\'ve told you he\'s not proper for the position. Speaker A: __________. A.I don\'t really agree with you B.I should have listened to you C.It doesn\'t matter.I trust him D.Thank you for being so helpful 47.Greg: Hey Merlin.I\'d like to ask you a question. Merlin: ____________. Greg: Well, I\'m thinking about going to Sweden.What\'s the best time to go? A.Yes, go ahead.B.Sorry, I\'m kind of busy. C.OK, what\'s up? D.Yeah, what\'s on your mind? 48.Woman: I need to buy a wedding gift for Jane and Desler. Man: Should we stop at the shopping center? Woman: _____________.The wedding\'s not until next week, but I won\'t have time later to get them anything. A.Won\'t be neceary B.I suppose so C.It\'s your call D.If you insist 49.Donald: Let\'s eat out, shall we? Mason: I\'m broke.I\'ve gone through my paycheck for the week already. Donald: Don\'t worry.___________. A.We can find a way B.Let\'s split the bill C.Just fast food D.It\'s my treat 50.Teacher: Richard, cla begins at 9, and you are late. Student: I know, but I mied my bus.I\'m sorry. Teacher: ___________.You have to be here on time. A.Don\'t mention it B.That\'s no excuse C.You needn\'t be D.No problem 英语: 1-5 CBBDC 6-10 DCADA 11-15 BACCB 16-20 ABCDB 21-25 BDCAA 26-30 DABAB 31-35 DABCB 36-40 CDACA 41-45 DACCD 46-50 BCBDB

第14篇:GCT考试单科模拟:英语篇(二)

Part One Vocabulary and Structure Directions: There are ten incomplete sentences in this part.For each sentence there are four choices marked A, B, C and D.Choose the one that best completes the sentence.Mark your answer on the ANSWER SHEET with a single line through the center.1.Many newcomers complain of the rapid _________ of life in Hong Kong.A.rate B.pace C.speed D.growth 2.To one’s bo, an employee should dre neatly, be ________ and show interest in the job.A.instant B.timely C.punctual D.quick 3.The girl will not become a nurse because she will faint at the __________ of blood.A.vision B.eyes C.sight D.view 4.The government_________ regulations that put this old city under protection.A.published B.iued C.discharged D.released 5.Unexpectedly the light __________ and we were left in darkne.A.bum out B.put out C.turned out D.went out 6.After a whole day’s heavy work, the old worker returned home,_________ .A.hungry and felt exhausting B.hunger and exhausted C.hungry and exhausted D.hungry and having been exhausted 7.The last half of the nineteenth century __________ the steady improvement in the means of travel.

A.has witneed B.was witneed C.witneed D.is witneed 8.She ___________ in the feet on her way home from work.A.was hurting B.is hurt C.hurts D.got hurt 9.Without computers, we ______ the tremendous medical advancement in the last few decades.A.would not make B.will not have made C.could not make D.couldn’t have made 10.___________ that Susan hadn’t dared to make a sound.A.So was he absorbed B.So absorbed he was C.So absorbed was he D.So he was absorbed Part Two Reading Comprehension Directions: In this part there are four paages followed by questions or unfinished statements, each with four suggested answers.Choose the one that you think is the best answer.Mark your answer on the ANSWER SHEET by drawing with a pencil a short bar acro the corresponding letter in the brackets.Questions 11—15 are based on the following paage: Some psychologists maintain that mental acts such as thinking are not performed in the brain alone, but that one’s muscles also participate.It may be said that we think with our muscles in somewhat the same way that we listen to music with our bodies.You surely are not surprised to be told that you usually listen to music not only with your ears but with your whole body.Few people can listen to music that is more or le familiar without moving their body or more specifically, some part of their body.Often when one listens to a symphonic concert on the radio, he is tempted to direct the orchestra even though he knows there is a competent conductor on the job.Strange as this behavior may be, there is a very good reason for it.One cannot derive all poible enjoyment from music unle he participates,

so to speak, in its performance.The listener “feels” himself into the music with more or le noticeable motions of his body.The muscles of the body actually participate in the mental proce of thinking in the same way, but this participation is le obvious because it is le noticeable.11.Some psychologists maintain that thinking is __________.A.not a mental proce B.more of a physical proce than a mental action C.a proce that involves our entire bodies D.a proce that involves the muscles as well as the brain 12.Few people are able to listen to familiar music without __________.A.moving some part of their body B.stopping what they are doing to listen C.directing the orchestra playing it D.wishing that they could conduct music properly 13.Body movements are neceary in order for the listener to _____________.A.hear the music B.appreciate the music C.enjoy the music fully D.completely understand the music 14.According to the selection, muscle participation in the proce of thinking is ______.A.deliberate B.obvious C.not readily apparent D.very pronounced 15.The best title for this selection is ____________.A.An Ear for Music B.Music Appreciation

C.How Muscles Participate in Mental Acts D.A Psychological Definition of the Thinking Proce Questions 16—20 are based on the following paage: During the summer seion there will be a revised schedule of services for the university community.Specific changes for intercampus bus services, summer hours for the cafeteria, the infirmary (校医院) and recreational and athletic facilities will be posted on the bulletin board outside of the cafeteria.Weekly movie and concert schedules which are in the proce of being arranged will be posted each Wednesday outside of the cafeteria.Intercampus buses will leave the main hall every hour on the half hour and make all of the regular stops on their route around campus.The cafeteria will serve breakfast, lunch, and early dinner from 7 a.m.to 7 p.m.during the week and from noon to 7 p.m.on weekends.The library will maintain regular hours during the week, but shorter hours on Saturdays and Sundays.The weekend hours are from noon to 7 p.m.All students who want to use the library borrowing services and recreational athletic, and entertainment facilities must have a valid summer identification card.This announcement will also appear in the next iue of the student newspaper.16.Which of the following is the main purpose of this announcement? A.To tell campus personnel of the new library services.B.To announce the new movies on campus this summer.C.To notify university people of important schedule changes.D.To remind the students to validate their identification cards.17.Specific schedule revisions for which of the following facilities are listed in this announcement? A.Athletic and recreational.B.Food and transportation.C.Bookstore and post office.D.Medical and audio-visual.18.Times for movies and concerts are not listed in this announcement because ______________.

A.film or concert occurs every Wednesday at 7 p.m.B.the full list would be too long C.films and concerts cannot be announced publicly D.the full list is not ready yet 19.According to the announcement which of the following is true of special summer hours for the library? A.It has them only on the weekends.B.It has them both weekdays and weekends.C.It has no special summer hours.D.The information is not available.20.According to the announcement, a validated identification card is required to __________.A.ride on intercampus buses B.read announcements in the cafeteria C.make use of the infirmary D.check books out of’ the library

Questions 21—25 are based on the following paage: The economy in the United States is heavily dependent on aluminum, a material widely used in the construction of buildings and in making such diverse things as cars, airplanes, and food containers.In 1979 Americans used over five million tons of new aluminum, and one and a half million tons of recycled aluminum.Some ninety percent of the bauxite (矾土) ore from which new aluminum is normally derived had to be imported to meet the demand.Poorer ores are abundant in the United States, however, and researchers at Purdue University may recently have found a way to obtain aluminum magnetically from these.Although aluminum is not attracted by ordinary magnets, under special conditions it becomes temporarily “paramagnetic”, or very weakly responsive to a magnetic field.This is achieved by immersing ore particles in water to which certain salts have been added and then

filtering the ore through steel wool in the presence of a strong magnetic field.It is hoped that this technique will reduce the amount of high-grade aluminum the United States must import.21.The paage focuses on _____________.A.construction and manufacturing industry in the United States B.new technique of obtaining aluminum from poor ores C.consumption and production of aluminum in the United States D.the annual import of high-grade aluminum into the United States 22.It is believed that a new technique to _________ will probably be found.A.recycle old aluminum products B.discover more bauxite ores C.make aluminum by putting ore particles under water D.take out aluminum magnetically from poorer ores 23.In the second paragraph the word “paramagnetic” means ___________.A.slightly magnetic B.a weak magnetic field C.a strong magnetic substance D.the under-water magnetic power 24.By immersing ore particles in water, ____________.A.we can get ore particles of equal size B.we can purify aluminum particles C.we can filter out aluminum D.we can improve the working conditions around 25.We understand from the paage that the United States ____________.

A.is rich in low-grade aluminum-beating ores B.is short of aluminum-making techniques C.has to import ninety percent of its annual need of aluminum D.has to produce fifty percent of the world’s supply to meet its annual aluminum demand Questions 26—30 are based on the following advertisements: Service Ad Profeional Typing Service announces a new location in Westside Mall, 1400 University Avenue acro from State University Student Union.We specialize in term papers, theses, and diertations typed to the specifications of the Graduate School of State University.Twenty-four-hour service for fifty pages or le.Forty-eight-hour service for more than fifty pages.Rates: $1 per page on regular paper $1.25 per page on cotton bond paper $ 0.25 extra for each carbon copy or a graph Hours: 8:00 a.m.—10:00 p.m.Monday—Friday 8:00 a.m.—4:00 p.m.Saturday Closed all day Sunday Call: 717-5415 26.What kind of person needs this service? A.A doctor.B.A policeman.C.The sick.D.A student.27.How much will it cost you if you want five regular papers and two graphs to be typed? A.$5 B.$1 C.$5.5 D.$6

28.If you have 20 pages to be typed, how long does it take to get them finished? A.More than 48 hours B.More than 24 hours.C.Within 24 hours.D.In 24 minutes.29.When does the typing service close on Wednesday? A.10:00 p.m.B.8:00 p.m.C.4:00 p.m.D.It’s closed all day.30.The main purpose of this paage is __________.A.to criticize B.to describe C.to advertise D.to celebrate Part Three Cloze Directions:

There are ten blanks in the following paage.For each numbered blank, there are four choices marked A, B, C and D.Choose the best one and mark your answer on the ANSWER SHEET with a single line through the center.As the plane circled over the airport, everyone sensed that something was wrong.The plane was moving unsteadily through the air, and 31 the paengers had fastened their seat belts, they were suddenly thrown forward.At that moment, the air-hoste 32 .She looked very pale, but was quite 33 .Speaking quickly but almost in a whisper, she 34 everyone that the pilot had fainted and asked if any of the paengers knew anything about machines or at least how to drive a car.After a moment’s 35 , a man got up and followed the hoste into the pilot’s cabin.

Moving the pilot aside, the man took his seat and listened carefully to the urgent instructions that were being sent by radio from the airport below.The plane was now dangerously close 36 the ground, but to everyone’s relief, it soon began to climb.The man had to 37 the airport several times in order to become 38 with the controls.Therefore the danger had not yet paed.The terrible 39 came when he had to land.Following information, the man guided the plane toward the airfield.It shook violently 40 it touched the ground and then moved rapidly along the runway and after a long run it stopped safely.

31.A.although B.while C.therefore D.then 32.A.showed B.presented C.exposed D.appeared 33.A.well B.still C.calm D.quiet 34.A.inquired B.insured C.informed D.instructed 35.A.hesitation B.surprise C.doubt D.delay 36.A.to B.by C.near D.on 37.A.surround B.circle C.observe D.view 38.A.intimate B.familiar C.understood D.close 39.A.moment B.movement C.idea D.affair 40.A.as B.unle C.while D.so Part Four Dialogue Completion Directions: There are ten short incomplete dialogues between two speakers, each followed by four choices marked A, B, C, and D.Choose the one that most appropriately suits the conversational context and best completes the dialogue.Mark your answer on the ANSWER SHEET by drawing with a pencil a short bar acro the corresponding letter in the brackets.41.Mother: Jimmy, what are you hiding behind your back? Let me see.Jimmy: It’s kitty.Please let me keep it.It’s a good kitty and will not bring you any trouble.Mother: __________ .I told you.No pets.It’ll make a me of this house.A.No way B.Not at all C.By the way D.On the contrary 42.Sophia: There’ll be an iue next month introducing your newly published book and I’d like to have an interview with you, if poible.Josh: An interview? I’d like to, but I’m afraid my schedule is all full.

Sophia: It won’t take you long, Mr.Josh.__________.A.One and a half hours will satisfy B.One and a half hours will do C.One and a half hours will go D.One and a half hours will help 43.Mother: Tom, it’s 7:30, you’re going to be late for school.Tom: I feel like another five minutes’ sleep.Mother: __________ .Get dreed.A.Come on! B.Fast! C.Hello! D.Anyway.44.Bus driver: Move to the rear of the bus, please.There are plenty of seats.Paenger: __________ ? Bus driver: Forty cents.Drop it in the box.A.What is the price, please B.What shall be the fee, please C.How much does it cost, please D.How much is the fare, please 45.Stephen: Well, hello, stranger! _________ Gordon: No, I went to California for a few weeks.Stephen: Oh, really? Where did you go? Gordon: Los Angeles.I stayed with my brother.A.Have you been away all these days? B.Have you moved or something? C.Haven’t seen you for ages! D.Nice to see you here.

46.Husband: Can I wait at the coffee bar? I feel ill at ease when you are picking things out.Wife: _______ I don’t want to shop alone.You can always give me advice, or enjoy looking at beautiful women.Husband: Don’t talk nonsense.

A.Please, honey B.All right, honey C.Excuse me, honey D.Sorry, honey 47.Lucia: Do you have any tiue, Polly? Polly: Here.___________.Lucia: Thanks.I have a terrible headache.It seems to be flu.A.What do you want to do? B.Can I help you? C.Are you all right? D.I’m glad to help.48.Tony: Will you please pa me the saltshaker, Bill? Bill: Sure.___________.Tony: Thank you.A.Give it to you B.Take it C.Here you are D.Hold it 49.Mother: Danny! Don’t make any sound when you have your soup.Danny: Yes, Mum.Father: ___________ He’s only a child.

A.I completely agree with you B.The noise is very disturbing C.Don’t be very cruel to him D.Don’t be so hard on him 50.Paenger: Taxi! (A cab stops and he goes in.) Cab driver: ___________? Paenger: King’s Hotel on North Street.

A.Where to go, sir B.To where, sir C.Where to, sir D.Go where, sir

1、答案B。pace of life意为“生活节奏”;rate意为“比率,速率”;speed意为“速度”;growth意为“发育,成长”。

2、答案C。instant意为“立刻的,即时的”;timely意为“及时的”;quick意为“(做事)迅速的”;punctual意为“守时的,准时的”。

3、答案C。at the sight of意为“看见”;view意为“眼界,视野”;vision是指人的视力或视野,引申为梦幻景象。

4、答案B。iue意为“发布,颁布,发行”;publish意为“出版,印刷”;discharge意为“释放出”;release意为“发布消息”。

5、答案D。burn out意为“烧光,烧完”;put out意为“熄灭,扑灭”;turn out意为“关掉,熄灭”;go out意为“熄灭,停止运行”。

6、答案C。从时态意义上讲,应该用过去分词,表示伴随情况,还应该考虑连词前后的平衡结构表达法问题。

7、答案C。根据主语可判断出时间为过去时。

8、答案D。“get+-ed分词”构成被动语态,get通常表示动作的结果而非动作本身。

9、答案D。without作“如果没有”解,表示一种与事实相反的假设,意思上相对于一个if引导的虚拟条件句。根据题中可看出without computer 是与过去事实相反的假设,因此主句应该用:would(should,could,might)+动词的完成时。

10、答案C。so为程度副词,置于句首时,句子一般倒装。当so作为程度副词置于句首时,它所修饰的形容词或分词一般紧跟其后。

11、答案D。本题考点在文章第一句中的mental acts such as thinking are not performed in the brain alone, but that one’s muscles also participate。原文中的not„alone, but„相当于考题里的as well as, 所以考生对意思相似相近的表达要清楚。

12、答案A。本题的依据句是第二段中的第二个句子Few people can listen to music that is more or le familiar without moving their body or more specifically, some part of their body.

13、答案C。本题依据关键在第三段第二句中的derive all poible enjoyment from music,这与试题中的enjoy the music fully意思相当。

14、答案C。本题的依据是短文中的最后一句,关键部分是this participation is le obvious,这与选项C、not readily apparent意思相当。

15、答案C。短文先指出思维中可能有类似听音乐时肌肉参与的情形;接着描述了听音乐过程中肌肉参与的情况;最后指出,肌肉在思维活动中的参与只不过没那么明显罢了。可见,全文主要讨论的是肌肉在思维活动中的参与,既不是专门讲思维,更不是专门说音乐。故C选项做标题合适。

16、答案C。本文讲的是“暑期校区各项服务安排变更情况”,首句即主题句为During the summer seion there will be a revised schedule of services for the university community.选项C符合题意。

17、答案B。本题属于细节题。要注意题干中的Specific schedule revisions,即“具体安排的修改”。第二段首句和第二句分别详细描述了公交车的发车时间、地点和食堂就餐时间,文中没有提到A.Athletic and recreational, C.Bookstore and post office和D.Medical and audio-visual.的具体情况。

18、答案D。本题的依据是文章第一段第三句话。关键部分是Weekly movie and concert schedules which are in the proce of being arranged,即“每周电影和音乐会计划还在安排当中”,故D为合理选项。

19、答案A。根据题干中的special summer hours for the library可以缩小寻找答案的范围,只寻找与“图书馆”有关信息即可。答案依据在第二段的第三句,其中but shorter hours on Saturdays and Sundays是关键部分。but shorter hours 表示special,与平时不同; Saturdays and Sundays指的就是weekends。因此A为正确选项。

20、答案D。本题答案依据是短文倒数第二句。句中提到“借书和娱乐健身”需要有效证明卡。注意题干里的validated和原文中的valid意思相同,即“有效的”。

21、答案C。本题考查的是对短文主旨大意的理解。首段说的是美国对铝的需求及其来源,第二段讲述了一种新的提取铝的方法。故C、consumption and production of aluminum in the United States最符合题意,A、B、D虽然提到与短文有关的内容,但都不够全面。

22、答案D。本题的依据是第一段最后一句,关键部分是Poorer ores,have found a way to obtain aluminum magnetically。

23、答案A。这是个词汇题。解这种题需要先找到该词所在的句子,答案往往从中可以读出。Paramagnetic在第二段首句,句子后半部分的or very weakly responsive to a magnetic field就是解释,故选A.slightly magnetic。

24、答案C。本题的依据是最后一句It is hoped that this technique will reduce the amount of high-grade aluminum the United States must import.显然,把矿粒进入水中的做法目的只有一个,即提取铝。故选C。

25、答案A。这是一个细节题,答案依据是第一段第四句Poorer ores are abundant in the United States„

26、答案D。这是一则打印服务广告,其中针对学生论文描述了服务内容和价格,可见服务对象是学生。

27、答案C。根据广告上的价格计算:$1 per page on regular paper,$ 0.25 extra for each carbon copy or a graph,5页regular paper需要$5, 2个graphs需要$0.5, 共计$5.5。

28、答案C。根据广告上的承诺:Twenty-four-hour service for fifty pages or le.20页的材料应该在24小时内打印完成。

29、答案A。按照广告上的服务时间:8:00 a.m.—10:00 p.m.Monday—Friday,周三应该是晚上10点停止服务。

30、答案C。这是一则针对学生的校园服务广告,包括服务地点、时间、内容和价格。

31、答案A。仔细阅读上下文,就能看出“and 31 the paengers had fastened their seat belts, they were suddenly thrown forward.”两个句子是对比关系,意思为“尽管旅客系着安全带,但大家还是被抛向前面”。

32、答案D。A选项正确说法应该为“showed up”,B和C选项意思和用法不符合语境;只有D选项正确。Appear意为“出现”。

33、答案C。应该注意but一词的意义在于引起一个相反的概念。“She looked very pale, but was quite 33 .”也就是说,所选择的单词应该是pale一词含义的反义词。所以用C选项最合适。全局意思为“尽管她脸色苍白,但她很镇静”。其他选项意思或用法有误。

34、答案C。A选项意思为“询问”;B选项意思为“保险”;C选项意思为“告诉”;D选项意为“指示”。只有C选项的意思和用法正确。

35、答案A。A选项意为“犹豫”,最符合本句话的场景。全句意思为“犹豫一会后,一个人站了起来跟服务员进了驾驶舱”。

36、答案A。close to属于固定搭配词组,意思为“接近”。其他选项的搭配不正确。

37、答案B。A选项意为“包围”;B项意为“绕圈子”;C和D选项的意思为“观察,看”。在这里可以判断,飞机是在机场上空绕了几个圈子。

38、答案B。A选项意思为“亲密的”;B项意为“熟悉”。本句话的含义是,飞机是在机场上空绕了几个圈子,目的是熟悉空管指示。

39、答案A。A选项的意思为“时刻”。全句意思是“可怕的一刻来临了,他要降落了”。其他选项的意思不正确。

40、答案A。注意本处的意思为“当„„时候”。A项as与C项while的意思都是“当„„时候”,但while指一个较长的时间过程,而as指事情发生的一刻。由于飞机触地touch the ground是一个短暂性动作,此处应选择as。

41、答案A。从Mother的回答看,她坚决不许Jimmy养猫。A No way 表示“绝对不行”,符合题意; B.Not at all意为“一点也不„„” C.By the way 表示转换话题; D.On the contrary意为“相反”,均不合适。

42、答案B。“一个半小时就行”的习惯表达用do,故选B。

43、答案A。对话中,母亲催促Tom快点起床。A Come on!表示催促,符合要求。

44、答案D。对话中乘客问车票多少钱。车票费用要用fare,而fee则表示“学费,入场费,手续费”等。

45、答案B。从下文的否定回答推测,问话人的话应该是Have you moved or something?即“你是搬家了还是怎么的?”

46、答案A。丈夫表示不愿意进商店买东西,而妻子极力劝他进去,因此我们选择A.Please, honey,表示请求。

47、答案C。Lucia跟Polly要纸巾,后来又说头疼,可能得流感了。Polly合理的反应是关切的询问,而不应该在没弄清情况时就说帮忙的话。故不选B,D。要纸巾显然是擦鼻子,A的提问是多余的。

48、答案C。递给别人所要的东西时说Here you are,意思是“给你”,其他表达不符合习惯。

49、答案D。母亲教育孩子喝汤不要弄出声响,从父亲的话听得出来,他嫌母亲的语气过于严厉了。所以不选A,B。C中的cruel使用不当。

50、答案C。出租司机问乘客“上哪儿?”英语的习惯表达是Where to,其他选项不正确。

第15篇:GCT考试英语强化训练模拟试题(一)

2009年GCT考试英语强化训练模拟试题

(一)

Part One Vocabulary and Structure

Directions:

There are ten incomplete sentences in this part.For each sentence there are four choices marked A,B,C and D .Choose the one that best completes the sentence.Mark your answer on the ANSWER SHEET with a single line through the center.

1.If only ______ I the books on the reading list.

A.read

B.have read

C.had read

D.could have read

2.The museum’s new building will have ______ the exhibition space as before.

A.three times

B.three times like

C.three times as

D.three times larger

3.When you are in your room, leave the door ______ so that your visitors do not have to knock.

A.open

B.opened

C.opening

D.being open

4.Mobile phones are proved to ______ with flight instruments and have a negative effect on flight safety.

A.interfere

B.disturb

C.interrupt

D.trouble

5.Retail sales volume in local urban and rural areas rose 57.8% and 46.8% __last year.

A.individually

B.respectively

C.correspondingly

D.accordingly

6.______ The science of medicine, progre has been very rapid lately, is perhaps the most important of all sciences.

A.to which

B.with which

C.in which

D.among which

7.It was the training that he had when he was ______ made him such a good manager.

A.has

B.later

C.which

D.that

8.This pair of glaes is similar to ______ you bought last week.

A.the one

B.the one what

C.the ones what

D.the ones

9.The terrible noise from that houseis almost ______ me mad.

A.mining

B.driving

C.setting

D.puffing

10.Peter likes watching TV ______ to the cinema.

A.more than to go

B.more than going

C.than going

D.rather than to go Part Two Reading Comprehension

Directions

In this part there are paages and one listing, each followed by five questions or unfinished statements.For each of them, there are four choices marked A,B,C and D.Choose the best one and mark your answer on the ANSWER SHEET with a single line through the center.

Questions 11-15 are based on the following paage:

Internet shopping is a new way of shopping.Nowadays, you can shop for just about anything from your armchair.Shopping on the Internet is becoming increasingly popular.In the United States people spent US$2.5 billion on Internet shopping in2003.

People can shop for a variety of products on the Internet.Physical products include items such as books, CDs, clothes and foods.These types of products are the most common purchases through the Internet.

Internet shopping offers a number of benefits for the shopper.The most important advantage is convenience.As the on-line shops are open 24\' hours a day you don\'t have to queue with other shoppers at the checkout counters.Secondly; it is easy to find what you are looking for on the Internet.Finally, it is often cheaper to buy goods throughthe Internet.

The main disadvantage of Internet shopping is that you cannot actually see the products you are buying or check their quality.Also, many people enjoy shopping in the city and mi the opportunity to talk to friends.

Internet shopping is sure to become more and more popular in the years ahead.It promises to change the way we buy all kinds of things from tonight\'s dinner to a new car.

11.The first paragraph indicates that _________.

A.there are many problems with on-line shopping.

B.people hate the on-line shopping in cities

C.it is veryinconvenient for on-line shopping

D.on-line shopping is becoming very popular

12.The phrase \"physical products\" in the second paragraph means

A.things that can beseen or felt

B.things useful for health care

C.things can help you keep fit

D.things connected with sports

13.The most important advantage about Internet shopping is ______ .

A.the speed the goods are delivered at

B.the reasonable priceand color

C.the quality ofthe goods

D.the convenience it brings to you

14.In the paage, the following disadvantages of Internet shopping are mentioned EXCEPT That ______

A.customers cannot actually see the products they are buying

B.customers cannot check the product\'s quantity they arebuying

C.many people mi the opportunity to talk to their friends

D.some people are worded about paying for goods using credit cards

15.The fifth paragraph is mainly about

A.people\'s enjoyment of a new way of shopping

B.Intermit companies\' finding of a new way for payment

C.there being various problems with Intermit shopping

D.serious disadvantages of Internet shopping

Questions 16-20 are based on the following paage:

Electronic books could change the way of reading, but people ought to consider their far-reaching impacts as well.\"The e-book promises to cause a slow tragedy on life as we know it,\" Jason Ohler, profeor of technology, warned the world.He weighed the advantages and disadvantages of e-book technology\'s impact on social relationships, the environment, the economy, etc.Before you read an e-book, consider the disadvantages.

He believed that the e-books increase harne of eye due to poor screen, replace a relatively cheap commodity with a more expensive one, and displace workers in print-book production and traditional publishing.E-books make it easy to share data, thereby threatening copyright agreements and reducing compensation of authors.

On the other hand, e-books save paper and trees, reduce the burden of the carrying and storing of printed books, promote self-sufficiency in learning, and make reading a collaborative experience online.They also create new jobs for writers and artists and encourage self publishing.

In final analysis, Ohler points out, e-books should gain society\'s approval if a few conditions are met, make them recyclable, solve the problem of eye fatigue, be sure the \"have-nots\" get the technology, and support e-book training in schools and busine.

16.What is the purpose of this writing?

A.To encourage people to use more e-books.

B.To help people how to read e-books effectively.

C.To talk about the impacts of e-books on society.

D.To make e-books more popular with the students.

17.One of the advantages ofe-books is ________.

A.Theycancreate new jobs for everyone.

B.They can save paper and trees.

C.They mayenhance the relationgships among people.

D.They may help protect eyes.

18.One of the disadvantages ofthe use of e-books is that ________.

A.they may promote self-sufficiency in learning.

B.they may pollute the environment.

C.they may encourage self publishing.

D.they may make some people lose their jobs.

19.What is Jason Ohler\'s attitude towards e-books?

A.Positive.

B.Indifferent.

C.Cautious.

D.Suspicious.

20.What is one of the problems which need to be solved?

A.Environmental protection.

B.E-book training in schools and busine.

C.Society’s approval of e-books.

D.Convincingpeople of using e-books.

第16篇:GCT考试英语大纲及试题结构

2012年GCT考试英语大纲及试题结构

一、考试目的

外语运用能力测试旨在考察考生目前所具备的实际英语水平、阅读英语能力和运用英语能力。通过英语词汇量、语法、阅读、理解、日常口语等内容的测试,了解考生使用英语的综合能力。

二、试题结构

1. 提型与题量

本部分共有50道题,考试时间为45分钟。试卷包括语法和词汇、阅读理解、完型填空、会话技能四部分

(1)语法和词汇

共有10道题,每题2分钟。前5道题为词汇题,后5道题为语法概念题。该部分时间大约为8分钟,分值为20分。

(2)阅读理解

共有4段独立的短文,每段短文约有150个英语单词。短文内容涉及政治、经济、历史、地理、文化、科技、人文、时事等。要求考生阅读每段短文后,回答5个问题。每个问题后有4个答案选项,其中1个选项为正确答案,要求考生选出正确答案。4段短文中,1篇较易,2篇难度适中,1篇较难。从近几年的真题来看,每年阅读理解最后一篇文章为应用文。该部分考试时间大约为21分钟,分值为40分。

(3)完型填空

提供一篇约240个单词的短文,在短文中有10个空白。每个空白有4个填空选项,其中1个为正确答案,要求考生选出正确答案。该部分考试时间大约为8分钟,分值为20分。

(4)会话技能

有10段英语简短对话,每段对话是不完整的,在每段对话后有4个答案选项,要求考生从中选出1个最符合情景和习惯用法的答案,使得整个对话通顺完整。该部分考试时间大约为8分钟,分值为20分。

2. 试题难易程度

试题由难分为3个等级,每套试题中容易、一般和较难的题目比例为3:5:2。

3. 试题评分标准

本部分试题满分为100分,每道题2分。考生须从每道题所列的A、B、C、D四个备选答案中选出一个正确答案,多选、不选或错选均不得分。

三、命题范围

英语使用能力考试命题范围的水平相当于四年制大学非英语专业毕业生应达到的水平。着重测试考生运用英语的能力,要求考生熟练掌握4000个以上的英语词汇和基本语法内容。考生需具有一定的阅读、写作和口头会话的能力,并具有一定的社会、生活和工作背景知识。(1)词汇和语法部分主要是测试考生的基本词汇和语法规则。

(2)阅读理解部分主要测试考生英语的阅读、理解、分析、判断能力。考生需要掌握一定的词汇量和具有一定的阅读速度以及知识背景。4段短文中,其中1段为科普内容,1段为英语应用文。

(3)完型填空主要测试考生对词汇和语法的掌握程度,语法内容包括冠词、名词、代词、形容词、副词、介词、连词、分词、不定式、从句、时态、语态、倒装和虚拟等。同时,考生还需要具有一定的阅读理解能力。词汇与语法的比例为1:2。

(4)会话技能主要测试考生使用英语进行日常会话的能力,这些会话涉及的是生活中的常见情景,测试考生理解会话的情景、把握对话人的角色以及掌握英语口语习惯用法的能力。

第17篇:GCT逻辑答案

2005年GCT入学资格考试逻辑推理能力试题 参考答案与解析 1.【答案】C。

【解析】假设C项是前提,即要知道一个命题的真实性,通过观察或实验证明为真即可。因此可推知题干中命题不能被证明为真,所以无法知道任何宗教命题的真实性。因为有了C项前提,所以使题干的推论更合乎逻辑。 2.【答案】D。

【解析】题干主要讲述了对待知识分子、工人和农民的态度,具有同等的重要性。A、B、C选项的陈述都不正确,因此答案为D。 3.【答案】C。

【解析】题干中说从皇帝的日记中不能看出皇帝的真实形态,所以只有将正史与野史结合起来,才能看出皇帝的真面目,即得出结论必须读野史。因此答案为C。 4.【答案】B。

【解析】假设B项为真,地价的上涨幅度与房价比起来是很小的,因此就不能得出题干中的断言:地价上涨是房价猛涨的罪魁祸首。A和D项虽然也说明地价的上涨与房价的上涨有关,但没有B项的反驳力度强。因此答案为B。 5.【答案】C。

【解析】题干的论证是:近几年因为政府加大投资才会出现经济稳定增长的态势,但这不表明是经济的良性循环。由此可以得出的结论是C项。 6.【答案】D。

【解析】由题干中陈述的条件我们可以推出前三项,但D项无法推出。 7.【答案】C。

【解析】本题考查的是一个充分必要条件的推理问题。由所给条件可判断C项为真。 8.【答案】D。

【解析】根据文意,我们可以判断上述论证依赖的假设正确的是D项。 9.【答案】B。 【解析】若B项为正确假设,即前提是仅仅忠实地记述历史事实的史学作品不是好的史学作品,因此可以得出好的史学作品必须突破那些僵化的历史真实观,真正触及到历史人物的灵魂,写出历史的本质真实来的论证。这样才使题意完整。其他假设都不合理。 10.【答案】C。

【解析】备选项中只有C是用充分条件得出的结论,充分条件未必能使结论成立。而其他三项都是用必要条件得出的正确结论。因此最不接近文中意思的只有C项。 11.【答案】A。

【解析】依据题意可知,如果A项假设为真,则可以说明文中的结论成立。 12.【答案】D。

【解析】D项的陈述正是对外部世界存在的一种很好的解释,因此加强了题干的论证。A项预先假定了外部世界的存在来说明外部世界肯定存在是无根据的;B选项也对题干的论证提出了怀疑;C项中海市蜃楼并非真实的实体,但我们确实看到了,因此得出与题干矛盾的结论。由上可知,只有D项为正确答案。 13.【答案】D。

【解析】题干中指出:必须采取扩张的财政政策,但这样做会带来什么样的利弊呢?这种做法是否可行呢?而D项的陈述正是对这一问题的回答,因此对评价这一论证最为重要。 14.【答案】D。

【解析】我们从文章的最后一句话可以推出,没有采取保护措施的兵马俑表面的生漆层会很快发生起翘和卷曲,造成整个彩绘层脱落,不能供研究使用,但不至于丧失所有的信息。有可能提供更多的信息。因此答案为D,其他推论都不正确。 15.【答案】B。

【解析】题干中已经表明,筹集更多的开发资金只有两种方法:直接贷款或预售商品房。现结果是开发商无法筹集到更多的开发资金,那么肯定是两种方法都行不通。因此另一个原因只有是B项。 16.【答案】D。

【解析】从文中可以直接推出文中论述所依赖的假设应是D项。其他假设都不成立。 17.【答案】A。

【解析】A项说大学排名对学校声誉与考生报考有很大的影响,与文中内容没有产生矛盾。B、C、D项内容都与题干存在不一致性,因此都驳斥了题干的论述。故答案为A。 18.【答案】B。 【解析】B项中的性别问题不是问题的根源所在,因此构不成对研究结论的质疑。A中如果受访者说的是假话,那么题干中的调查就不可靠。C指出了是否有生活习惯方面的差异的重要遗漏,这也可能导致调查结果的不可靠。D也指出了患癌症的另一个原因是由于遗传性的原因而非生活习性的原因,也使研究结论不成立。因此正确答案为B。 19.【答案】A。

【解析】要想使题干的结论成立,必须说明其成立的原因。备选项中只有A项很好的验证了这一结论,因为如果知道了其他很少食用这种蔬菜的群体中患病的比例的大小,经过对比,就可以知道这种蔬菜是否到底有没有抵抗和预防患这种癌症的作用。 20.【答案】C。

【解析】由题干的陈述为真,可以得出如果没有拥有私有财产的合法权力的话,那么私有化就是一句空话。即C项得到了最有力的支持。 21.【答案】B。

【解析】从题干中得出的结论是:经济发展和经济不发展。经济发展了,自然环境必然要恶化;经济不发展就会停滞。因此答案只有选B。 22.【答案】A。

【解析】由题意可知,分红和融资不能同时进行,分红就不需要融资,融资就不能分红。据此可知不可能为假的只有A项。 23.【答案】D。

【解析】A、B、C项都没有违背分红和融资不能同时进行的原则,而D项是两都同时进行,因此不正确。 24【答案】C。

【解析】A、B、D选项都是解释说明为什么海南人自己不信赖自己的原因,只有C项无解释说明的作用,因此答案为C。 25.【答案】B。

【解析】题干的论证是权力是使人堕落和道德沦丧的最永恒、最活跃的因素,因此应限制和防范权力的滥用。如果权力不会使人堕落和道德沦丧的话,就不必要限制和防范权力的滥用。因此B项是对上述推理最强的支持。

26.【答案】C。

【解析】A选项不符合日常购物的习惯;B选项没有解释1 000亿元资金的流向问题的根源;D选项考虑是否购买股票或基金,并不等于1 000亿元资金都已经购买了这些股票和资金,还有很大的不确定性;因此这三项都不正确。C选项说因为民间贷款使得借贷利率远高于银行存款利率,由于追求利润的动力使得1 000亿元资金外流,这就很好的解释了1 000亿元资金流向的根源问题,因此答案为C。 27.【答案】A。

【解析】A选项正确,题干的意思就是在说破坏一个好声誉比消除一个不好的声誉更容易。B、C、D选项都不能得到题干的支持。 28.【答案】C。

【解析】这段话隐藏的一个条件是,皇帝故意让官员贪污,因此来控制他,否则,没有其他办法来控制。换句话说就是迫使官员贪污就是皇帝控制官员的唯一的方法。故选C。 29.【答案】A。

【解析】假设购买商品房的费用/平均家庭预算费用=50%,如果保持比例不变,则平均家庭预算费用增加了25%的话,那么购买商品房的费用也一定增加了25%。而新商品房的平均价格每平方米增加了25%,那么购买的面积是不能变的,否则就会超出购买商品房的费用也一定增加了25%这个比例。因此只有假设A为真才能使题干的陈述成立。 30.【答案】A。

【解析】B、C、D三项都未说明题干中所论述的原因和结果之间的联系,因此不能支持题干的结论。A项说相似的结果(呻吟和脸扭曲)一定有相似的原因(有痛感),因此很好的解释了文中的结论。 31.【答案】C。

【解析】备选项中只有C选项详尽的体现了其含义。 32.【答案】C。

【解析】文中的陈述最有力的支持了C项的结论,否则有了股份的话,企业的利润多了,职工的那一份利润也会增加。 33.【答案】D。

【解析】经过比较过去的“我应该干什么?干什么我能赚钱?”和现在的“干什么我能成为最好的?”,可以得出这样的结论:过去因为行业有好坏之分,所以才会选方向。而加入WTO之后,所有行业不再有好坏之分,从事各行各业都行,关键是怎么能成为最好。即D为正确答案。 34【答案】A。

【解析】很明显题干中论述的缺陷是没有评判的标准,因为所有评判并非是以自己的好坏和对错为标准的,如果没有一个标准的话,就无所谓对错好坏之分。所以正确答案为A。 35.【答案】B。 【解析】已知题干中表明最优行动准则的优势就是它能采纳弹性较大的标准,那么前提应该是采纳弹性较小的标准不能发挥最优行动准则的优势,这样才能使论证更合乎逻辑。即答案为B。 36.【答案】D。

【解析】从这段文字中的“坚持”和“抵抗”可以判断,A、B、C三个选项都是对文化的阐释,因此都可以从题干中推出,只有D不能从中推出。 37.【答案】B。

【解析】题干的论证是我国的各种制度不是要破坏亲情,而是要把亲情激发以最高点。而B项的内容直接对此论证进行了反驳,因此严重的消弱了题干论证。 38.【答案】D。

【解析】若D项正确,则说明中国的税制以商品税为主,一个人只要购买并消费商品,就向国家交了税。由此可见,纳税跟起征点的高低并没有多大的关系。从而对起征点太高就剥夺了低收入者作为纳税人的荣誉的说法提出了质疑。 39.【答案】A。

【解析】A选项的陈述与题干中的论述意见一致,当然就起不到消弱作用了。其他选项都能消弱题干结论。 40.【答案】A。

【解析】依据题干的论述可知,一个人如果知道自己活的时间比较长的话,他肯定不会去偷盗而选择做好事,也不会去冒因为偷盗而被抓住的危险。所以A项的假设条件是正确的,即一个人决定去偷盗时,肯定知道自己还能活多久。 41.【答案】B。

【解析】结合已给出的条件,可推知同时满足两组条件的只有B项。

42.【答案】B。

【解析】此题用代入法。替换F或L不符合条件(3);替换H不符合条件(5);只有替换G不违反任何给出的条件。即答案为B。 43.【答案】D。

【解析】已知K和J必须在一组,因此K和J不可能在任一组。由条件(4)可知,M必须在装订组里,其余的员工为G、H、L、F或G、H、L、K中的元素组合,要满足尽可能多的共同员工条件,则两组中的共同成员必是G、H、L。由此不难看出装订组的成员比印刷组的成员多一名M。即选项D为真。

44【答案】D。 【解析】由题意知,K和J不能分开,因此两组都有K和J。又因印刷组正好有三个成员,所以必是J、K、L。因为M至少是这两个组中的成员之一,所以装订组的成员至少必有J、K、M,其他陈述都不正确。 45.【答案】C。

【解析】由上分析可知,在满足条件的同时,两组中的共同成员最多有六名。 46.【答案】D。

【解析】根据条件,列出以下形式: (1)G/H→前位; (2)K→中位; (3)L→1队; (4)PK≠N; (5)P≠Q; (6)H=2→Q=1中位

由比不难看出不违反条件的只有选项D。 47.【答案】A。

【解析】只有A项是1队可接受的安排,其他选项都违反题干条件。 48.【答案】A。

【解析】同上,只有A项是正确的安排。 49.【答案】D。

【解析】代入法。把选项代入推出与题干不相符即不正确,此题的正确答案为D。 50.【答案】C。

【解析】解法同上,正确选项为C项。

第18篇:GCT考试大纲

2012年GCT考试数学大纲

一、考试目的

数学基础能力测试,旨在考察考生所具有的数学方面的基础知识、基本思想方法,考察考生逻辑思维能力、数学运算能力、空间想象能力以及运用所掌握的数学知识和方法分析问题和解决问题的能力。

二、试题结构 1.题量与题型

本部分共有25道题,考试时间为45分钟。试卷包含算术题、代数题、几何题、一元微积分题和线性代数题等五部分,每部分各占20%,均为单项选择题。 2.试题难易程度

试题难度分为:容易、一般、较难三个等级,在每套试题中,容易题、一般题和较难题的题量之比约为1:4:1。 3.试题评分标准

本部分试题满分为100分,每道题4分。考生须从每个问题所列出的A、B、C和D四个备选答案中选出一个正确答案,多选、不选或错选均不得分;所选答案均为A或B、C、D的答卷,一律视为废卷。

三、命题范围

数学基础能力测试的命题范围主要包括算术、代数、几何、一元微积分和线性代数的基础知识,及其在日常生活、科学研究和实际工程中的应用。要求考生对所列数学知识内容有较深刻的理性认识;系统地掌握数学知识之间的内在联系;通过举例、解释、分析、推断以解决相关问题;运用相关知识和逻辑推理方法分析、解决较为复杂的或综合性的问题。 1.数学基础能力测试的知识要求

数学基础能力测试所涉及的知识有:算术、代数、几何、一元微积分和线性代数。 (1)算术

数的概念和性质,四则运算与运用。 (2)代数

代数等式和不等式的变换和计算。包括:实数和复数;乘方和开方;代数表达式和因式分解;方程的解法;不等式;数学归纳法,数列;二项式定理,排列,组合和概率等。 (3)几何

三角形、四边形、圆形以及多边形等平面几何图形的角度、周长、面积等计算和运用;长方体、正方体以及圆柱体等各种规范立体图形的表面积和体积的计算和运用;三角学;以及解析几何方面的知识。 (4)一元微积分

① 函数及其图形:集合,映射,函数,函数的应用。

② 极限与连续:数列的极限,函数的极限,极限的运算法则,极限存在的两个准则与两个重要极限,连续函数,无穷小和无穷大。 ③ 导数与微分:导数的概念,求导法则及基本求导公式,高阶导数,微分。 ④ 微分中值定理与导数应用:中值定理,导数的应用。

⑤ 积分:不定积分和定积分的概念,牛顿—莱布尼兹公式,不定积分和定积分的计算,定积分的几何应用。 5)线性代数

① 行列式:行列式的概念和性质,行列式按行展开定理,行列式的计算。 ② 矩阵:矩阵的概念,矩阵的运算,逆矩阵,矩阵的初等变换。

③ 向量:n维向量,向量组的线性相关和线性无关,向量组的秩和矩阵的秩。 ④ 线性方程组:线性方程组的克莱姆法则,线性方程组解的判别法则,齐次和非齐次线性方程组的求解。 ⑤ 特征值问题:特征值和特征向量的概念,相似矩阵,特征值和特征向量的计算,n阶矩阵可化为对角矩阵的条件和方法。 2.数学基础能力测试的能力要求 (1)逻辑推理能力

对数学问题进行观察、比较、分析、综合、抽象与概括;能用演绎、归纳和类比进行推断。 (2)数学运算能力

根据数学的概念、公式、原理、法则,进行数、式、方程的正确运算和变形;通过已知条件分析,寻求与设计合理、简捷的运算途径。 (3)空间想象能力

根据数学问题的条件画出正确的图形,并根据图形想象出直观形象;能对图形进行分解、组合与变形。 (4)综合思维能力

理解和分析用数学语言所表述的问题;综合应用数学的知识和思想方法解决所提出的问题。

2012年GCT考试语文大纲/试题结构

一、考试目的

语言表达能力测试,旨在以语文为工具,测试考生的知识积累与语言表达能力。通过考生对字、词、句、篇的阅读与理解,考察其掌握自然科学、人文与社会科学知识的程度,以及运用语言工具表达知识的能力。

二、试题结构 1.题量与题型

本部分共有50道题,考试时间为45分钟。试卷包含选择题15题、填空题15题和阅读理解题20题。选择题部分为四选一型的单项选择题;填空题部分要求从四个答案中选择一项正确的答案填入题目所列空缺处,使试题内容完整;阅读理解部分提供5篇短文,每篇短文有4道题,要求根据文章提供的背景材料内容和线索,从四个选项中选择一个正确的答案填入,使答案与背景材料的内容相吻合。

2.试题难易程度

试题由易到难分为3个等级,每套试题中容易、一般和较难的题目比例为3:5:2。通过问题中所涉及的知识,测试考生应用语言进行观察、分析和解释问题的基本思想方法与能力,既测试考生知识积累的宽广度,又测试考生思维的敏捷性。

3.试题评分标准

本部分试题满分为100分,每道题2分。考生须从每道试题所列的A、B、C、D四个备选答案中选出一个正确答案,多选、不选或错选均不得分;所选答案均为A或B、C、D的答卷,一律视为废卷。

三、命题范围

本部分的知识背景涉及自然科学、人文与社会科学知识,包括哲学、经济学、法学、教育学、文学、历史学、理学、工学、农学、医学、军事学、管理学等学科门类。在测试考生知识面宽广程度的基础上,注重对考生在学习与实践中形成的思想方法的测试,注重对考生获取知识和表达能力的测试。 1.语言表达能力的层级

语言表达能力由低到高分为四个层级,依次是识记、了解、理解和应用,且高一级的层次要求覆盖低一级层次的要求。

(1)识记:要求对所列知识内容进行初步识别和记忆。

(2)了解:要求对所列知识内容作初步的、感性的认识,知晓有关内容,并能初步应用于有关的语言表达中。 (3)理解:要求对所列知识内容有较深刻的理性认识,能够进行解释和推断,并能运用语言解决有关问题。 (4)应用:要求系统地掌握知识的内在联系,能运用所列知识分析和解决较为复杂的或综合性的问题。 2.语言表达能力的要求

(1)识记与人类进步、社会发展和日常生活紧密相关的科学常识。 (2)了解各门科学的常用术语和表达形式,以及新成果和新术语。

(3)理解各门科学的基本概念、一般原理和普遍规律,并能够阅读记叙文、说明文、议论文等文体的短文,理解文中重要的词语,解释文中重要的句子,进行文章、段落的概括与归纳。 (4)应用科学知识去正确地解释和说明有关现象和问题,对已学知识具有综合、重组和转换的能力,且具有敏捷地接受新知识的能力。

2012年GCT考试逻辑大纲/试题结构

一、考试目的

逻辑推理能力测试,旨在考察考生掌握和运用逻辑分析方法的能力。运用给出的信息和已掌握的综合知识,通过理解、分析、综合、判断、归纳等过程,引出概念、寻求规律,对事物间关系或事件的走向趋势作出合理判断与分析,确定解决问题的途径和方法。

二、试题结构 1.题量与题型

本部分共有50道题,考试时间为45分钟。试题均为单项选择题,包括一题一问和一题多问两种类型。一题一问为给出相应的条件,完成一个问题的回答;一题多问则为给出一个完整的条件,完成几个问题的回答。 题目分为六类: (1)由前提确定结论型

题干所给已知条件是前提,要求考生在备选答案中选定合乎逻辑的结论。 (2)由结论寻找前提型

题干所给的是结论,要求考生在备选答案中寻找能得到此结论的前提。 (3)加强前提型

题干中有前提有结论,但题干中的前提尚不足以推出结论,要求考生在备选答案中寻找补充前提,以得出题干中的结论。 (4)反驳型

前面三种类型都是用推理进行证明。原题干中的前提真,或者经补充前提而使前提充分真,那么,可以演绎证明结论必然真,或者归纳证明结论非常可能真。由结论真寻找前提也是一种证明。反驳可以反驳论题(推理的结论)、反驳论据(推理的前提)和反驳论证方式(推理形式)。 (5)类比型

主要有:推理形式的类比、逻辑错误的类比和逻辑方法的类比。 (6)语义分析型

这类题目要求考生对日常语言表达的较为复杂的内容和含义有敏捷而准确的理解、分析和推理能力。 2.试题评分标准

本部分试题满分100分,每道题2分。考生须从每个问题所列的A、B、C、D四个备选答案中选出一个正确答案,多选、不选或错选均不得分;所选答案均为A或B、C、D的答卷,一律视为废卷。

第19篇:GCT考前辅导

2012年GCT辅导(1)

一、关于“李约瑟之谜”

二、如何进行逻辑考前辅导

1、

2、

3、偏重讲逻辑 偏重讲习题 两者结合型

我的看法:赞同3

“授人以鱼”与“授人以渔”

照顾大多数

附: 协作精神与竞争意识

“炮弹”与“青蛙’

三、MBA、MPA、GCT入学考试的源流

1、申请美国大学研究生院所要求的三大标准化考试——GRE、GMAT和LSAT

2、出题背景及考题分类(此处内容可作为中篇 论证推理的参考材料)

(1)出题背景

不需要任何学科的特定知识来回答问题,并且不能借助自己所熟悉的学科知识来理解问题,而应当从逻辑推理的角度来回答问题。

(2)逻辑推理题考查内容及题目的类型

逻辑推理题主要考查以下推理能力:1)进行论证的能力;2)评价论述的能力;3)形成或评价行动方案的能力。按照美国ETS命题人员的说法,逻辑推理题主要考查考生在以下三个方面能否进行有效推理的能力。

考查内容

一、论点构建(Argument construction)

这一方面的问题主要让你去识别或找到:

(一)论述的基本结构(The basic structure Of anargument)

(二)正确得到的结论(Properly drawn conclusion)

(三)基于的假设(Underlying aumption)

(四)被强有力支持的解释性假说(Well-supported explanatory hypotheses)

(五)结构上相似的论点的平行结构(Parallels between structurally similar arguments)

二、论点评价(Argmnent evaluation)

这一方面的问题主要让你在分析既定的论点基础之上去识别:

(一)加强或削弱既定论点的因素(Factors that would strengthen,Orweaken,the given argument)

(二)在进行论述时所犯的推理错误(Reasoning errors committed in making that argtanent)

(三)进行论述所使用的方法(The method by which the argument proceeds)

三、形成并且评价行动方案(Formulating and evaluating a plsn of action)

这方面的问题主要让你去识别:

(一)不同行动方案的相对合适性、有效性或效率(The relative appropriatene,effectivene,or effi—ciency of different plans of action)

(二)加强或削弱拟议行动方案成功可能的因素(Factors that would strengthen,or weaken,the prospects of succe for a proposed plan of action) (三)拟议行动计划所基于的假设(Aumptions underlying a proposed plan of action) 题目的类型

根据逻辑推理题的考查内容,我们认为熟悉逻辑推理的主要题型对你大有裨益,下面是我们在考试中将遇到的十一类逻辑推理题。

一、假设(Aumption):

这类考题主要考查我们识别根据什么前提得出论点的能力,

二、支持(Support):

这类考题主要考查我们识别一种附加事实信息支持论点的能力,

三、反对(Weaken):

这类考题主要考查我们识别一种附加事实信息反对论点的能力,

四、评价(Evaluation):

这类考题主要考查我们评价论点的能力,

五、推断(1nference):

这类考题主要考查我们通过作者明确的表述看出其含义的能力,

六、结论(Conclusion):

这类考题主要考查我们根据文章中的论据能提出什么合乎逻辑的主张的能力,

七、中心思想:

这类题主要考查我们理解文章要点的能力,

八、解释(Explain):

这类考题主要考查我们解释某个现象、结果或缓解某种矛盾的能力,

九、应用:

·

这类考题主要考查我们将一个论点的指导性原则用于另一论点的能力,

十、技法:

这类考题主要考查我们识别论点的结构方法或技法的能力,

十一、划线句子作用:

这类考题主要考查我们论点构建中的某句话对结论或前提是否起作用或起什么作用的能力。(MBA题无)

针对中国考生的思维特点,作者将考题分为假设、支持、反对、评价、归纳(包括推断、结论、中心思想等)、解释、逻辑应用与技法(包括Flaw、应用、技法、划线句子作用等)七大类。5-7

3、逻辑推理题四大出题原则及三大解题步骤

(1)四大出题原则

(一)不需专业背景知识原则(收敛思维原则)

(二)Which Of the following原则

(三)if true原则

(四)五中选二的原则(The rule of 2 out of 5)(GCT为四中选二原则) 参见教科书P4-5 (2)三大解题步骤

(一)读问题,明晰问题目的,从而迅速确定解题方向。

(二)读段落,根据不同的问题目的确定不同的解题重点。

(三)找答案,注意有些题型可调整看选项的顺序。

四、GCT试题三大类型

1、形式化推理

基础

分析推理

特色

2、非形式化推理

主流

五、时间安排、策略

三个阶段

第一阶段:系统复习(学习)

大致时间6(7)——9月初

第二阶段:训练提高

大致时间 9月上旬——10月上旬

选出最近的考题进行详解,应在参加考试前两周完成,不宜过早做题,因为其重要性之一在于考前的考场气氛的训练。最好的使用时间为考试前50天至考试前10天,以5天一套考题的训练量为宜。 第三阶段:模拟冲刺

大致时间10月中旬——考前(一般为10月20日左右)

个别

逻辑题目

特殊

逻辑试卷

一般

全科试卷

具体答题策略以后详讲

近代国学大师王国维说过,古往今来的大学问家都要经历三种境界,三种境界可用三句词的意境概括, 其一

“昨夜西风凋碧树。独上高楼,望尽天涯路”,

其二

“衣带渐宽终不悔,为伊消得人憔悴,”

其三

“众里寻他千百度,蓦然回首,那人却在,灯火阑珊处。”

这也是我们复习应考过程中会经历的三个境界 立志——悲壮

攻关——艰(痛)苦

3 成功——快乐

逻辑概述

p1-3

上篇

形式推理

补充:绪论

重点:思维形式的结构

p5

例题:

请指出下列各题中具有共同逻辑形式的判断或推理,并用公式表示之:(1—7)

1、所有的商品都是有价值的。

2、所有的金属都是导电的。

3、所有的偶数都是能被2整除的。

4、如果天下雨,那么地面湿。

5、如果他患肺炎,那么他发烧。

6、所有的偶数都是能被2整除的,

8是偶数;

所以,8是能被2整除的。

7、所有的金属都是导电的,

铜是金属;

所以,铜是导电的。

8、世界级的马拉松选手每天跑步都不超过6公里。因此,如果一名选手每天跑步超过6公里,他就不是一名世界级马拉松选手。

以下哪项与上文推理方法相同?

(A)跳远运动员每天早晨跑步。如果早晨有人跑步,则他不是跳远运动员。

(B)如果每日只睡4小时,对身体不利。研究表明,最有价值的睡眠都发生在入睡后的第5小时。

(C)家长和小孩做游戏时,小孩更高兴。因此,家长应该多做游戏。

(D)如果某汽车早晨能起动,则晚上也可能起动。我们的车早晨通常能起动,同样,它晚上通常也能起动。

(E)油漆三小时之内都不会干。如果某涂料在三小时内干了,则不是油漆。

解析

8、可以把题干分析为:

如果他是一名世界级的选手,那么他每天跑步不超过6公里。因此,如果他每天跑步超过6公里,那么他就不是一名世界级的选手。用“p”表示“他是一名世界级的选手”,

“q”表示“他每天跑步不超过6小时”,则“他不是一名世界级的选手’’可以表示为“非p”,

“他每天跑步超过6小时’’可以表示为“非q”,

这样,题干就具有如下的逻辑形式:

如果p,那么q;所以,

如果非q,那么非p 再分析试题中给出的五个答案,其中(A)、(B)、(C)显然属于案外案即明显地与题意要求无关的选项;(D)具有以下的形式:

如果p,那么q;所以,如果p,那么q。 与题干的逻辑形式不同,可予以排除;

而(E)可以分析为:

如果某涂料是油漆,那么它三小时之内不会干。因此,如果某涂料三小时之内干了,那么它不是油漆。显然,答案(E)具有

“如果p,那么q;所以,如果非q,那么非p’’的逻辑形式,与题干的逻辑形式完全相同,所以,正确的答案应该是(E)。

这种从逻辑形式上分析和把握问题的方法,可以使我们十分准确地寻找到正确答案。

需要说明的是,对于初学者来说,掌握逻辑常项和逻辑变项,并能熟练地用它们刻画一个思维过程的形式结构,是要化一些功夫的。但只要循序渐进,随着所学的内容的不断深化,辅之以必要的练习,那是完全可以办到的。

附:

1. 哪一个运动员不想出现在奥运会的舞台上,并在上面尽情表演?

如果以上陈述为真,以下哪项陈述必定为假?

A 所有美国运动员,如游泳选手菲尔普斯,都想在奥运会的舞台上尽情表演。

B.有的牙买加运动员,如短跑选手博尔特,想出现在奥运会的舞台上。

C.中国 110 米跨栏选手刘翔不想出现在奥运会舞台上,并在上面尽情表演。

D.任何一个人,只要他是运动员,他都想出现在奥运会的舞台上。

(2008年GCT逻辑试题p290)

2、黄铜不是金子,黄铜是闪光的,所以有些闪光的不是金子。

下述哪个推理的结构与上述推理最为相近?

A、宗教不是真理,真理是要受实践检验的,因此,宗教都是不要受实践检验的。

B、宝钢没有亏损,宝钢是国有大型企业,因此,有些国有大型企业没有亏损。

C、鲁迅是文学家,鲁迅是思想家,因此有些思想家是文学家。 D、坏人都攻击我,你攻击我,所以你是坏人。

E、金属都是导电的,植物纤维不导电,因此植物纤维不是金属。

GCT经典题目

形式化推理

1*2003GCT-4 有些台独分子论证说:凡属中华人民共和国政府管辖的都是中国人,台湾人现在不受中华人民共和国政府管辖,所以,台湾人不是中国人.

以下哪一个推理明显说明上述论证不成立?

A.所有成功人士都要穿衣吃饭,我现在不是成功人士,所以,我不必穿衣吃饭.

B.商品都有使用价值,空气当然有使用价值,所以,空气当然是商品.

C.所有技术骨干都刻苦学习,小张是技术骨干,所以,小张是刻苦学习的人.

5 D.犯罪行为都是违法行为,违反行为都应受到社会的谴责,所以,所有犯罪行为都应受到社会谴责 .

2*2003GCT-41 韩国人爱吃酸菜,翠花爱吃酸菜,所以,翠花是韩国人。

以下哪个选项最明确地显示了上述推理的荒谬?

A.所有的克里特岛人都说谎,约翰是克里特岛人,所以,约翰说谎。

B.会走路的动物都有腿,桌子有腿,所以,桌子是会走路的动物。

C.西村爱翠花,.翠花爱吃酸菜,所以,西村爱吃酸菜。

D.所有金子都闪光,所以,有些闪光的东西是金子。

3*2011GCT-14 2010年,卫生部推出新的乳业国家标准,将原奶蛋白质含量由原来的2.95%降至2.8% 新标准不升反降,引发了一片质疑。某业内人士解释说:如果我们的牛奶检测标准把蛋白质含量定的太高,奶农为了达标就会往奶里添加提高蛋白质检测含量的东西,如三聚氰胺。2008年的三聚氰胺事件就说明原来的标准太高了。

以下哪项推理含有与该业内人士的推理相同的逻辑错误?

A.真正的强者都不惧怕困难或挑战,赵涛害怕困难或挑战,说明赵涛不是真正的强者。

B.如果台风在海口登陆,飞往海口的航班就会被取消。现在飞海口的航班没有被取消,

说明台风没有在海口登陆。

C.要是铁路部门的管理存在漏洞,铁路运输就会出事故。7.23温州动车事故就说明铁

路部门的管理存在漏洞。

D.仅当人们信任一个慈善机构时才会向该机构捐款,所以,得不到捐款的慈善机构一定是丧失了公众的信任。

*2003GCT-33 美国在遭受“9.11”恐怖袭击后采取了这样的政策:“要么与我们站在一起去反对恐怖主义,那你是我们的朋友;要么不与我们站在一起,那你是我们的敌人。”下面哪一项与题干中的表达方式不相同?

A.有一则汽车广告:“或者你开凯迪拉克,那么你是富人;或者你根本不开车,那么你是穷人!”

B.以足球为职业的人只有两种命运:要么赢,那你是英雄,面对鲜花、欢呼、金钱、美女;要么输,那你是孬种、笨蛋,面对责难、愤怒、谩骂,打落牙齿往肚里吞.

C.如果一位教授有足够的能耐,他甚至能够把笨学生培养合格;因此,如果他不能把 笨学生培养合格,就说明他的能耐不够大。

D.要么你做一个道德高尚的人,那你就无私地贡献自己的一切;要么你做一个卑鄙的人,那你就不择手段地谋私利。

第一章 概念与定义p6-13 重点:

1、概念的内涵和外延

2、概念外延间的关系

3、定义

第20篇:GCT考试。语文

2012年GCT考试语文真题(A卷)及参考答题,方便2013年考生参考。 第一部分 语言表达能力测试 (50题,每题2分,满分100分)

一、选择题

1.下面各组词语,没有错别字的一组是 A.船坞 沉寂 人才倍出 浑水摸鱼 B.惊垫 没落 干均一发 无人问津 C.挑衅 振动 改弦更张 味同嚼蜡 D.匮乏 坚韧 迥迥有神 百战不殆 2.下面加点的词,意义相同的一组是

A.①整个建筑建造精美,典雅大方,又带有时代气息。 ②易玛活泼大方,总是有说有笑,是个令人愉快的姑娘。 B.①临时政府实力单薄,根本无法控制首都以外的地区。

②有些章节内容单薄,一些问题虽提出,但未能展开充分的论证。 C.①学员们集中精力,端正学风,取得了比较好的学习成绩。 ②南洋杉的树形端正、细长,姿态秀丽,是世界著名的观赏树木。 D.①张正一口纯正流利的英语配音,给观众留下了深刻的印象。

②政府正努力建设更多的知识园地,给孩子们以清雅纯正的文化环境。 3.下面各句,语义明确、没有歧义的一句是

A.他跟我去天津开过学术研讨会。 B.这些人连厂里的领导都不认识。 C.他曾先后在深圳、广州搞调研。 D.筐中苹果余下的一半给了老王。 4.下面各句,没有语病的一句是

A.那一刻,我们几个第一次看到他露出的真正属于破子般的灿烂笑容。 B.机动车进胡同,对机动车噪声是否只有被动接受,居民们早有微词。 C.我们的教育工作者积极发现人才、培养人才,这本身就是一种贡献。 D.殖民时代的业余运作方式,直接导致了澳门低忐平的竞技体育运动。 5.下面各句,加点成语使用不恰当的一句是

A.缅甸和我国接壤的地区,也就是臭名昭著的"金三角"地区,鸦片种植曾一度十分猖獗。

B.汽车越来越时尚化,连最保守的德国人都在汽车外形设计上做出了令人张口结舌的改变。

C.亚洲杯决赛的前30分钟,中国足球队的后防线险象环生,不断出现被对方突破的情形。

D.欧元下跌狂潮不仅使己陷入金融危机的欧洲经济雪上加霜,也给我国造成了负面影响。

6.下面诗句中,"锦书"一词意思与其他三句差别较大的一句是 A.青家曾无尺寸归,锦书多寄穷荒骨。(陈陶《关山月?) B.云中谁寄锦书来?雁宇回时,月满西楼。(李清照《一剪梅?) C.山盟虽在,锦书难托,莫!莫!莫!(陆游《钗头凤?) D.上元锦书传宝字,王母琼箱荐金约。(王勃《七夕赋?) 7.下面女性形象,依次出现在《西厢记??桃花扇》和《长生殿》中的是 A.李香君 杨玉环 崔莺莺 B.崔莺莺 李香君 杨玉环 C.崔莺莺 杨玉环 李香君 D.杨玉环 崔莺莺 李香君 8.下面字母词与汉语释义不符的一组是

A.①GOP:国民生产总值 ②GNP:国内生产总值

B.①NBA:美国职业篮球联赛 ②FIFA:国际足球联合会 C.①APEC:亚太经济合作组织 ②OPEC:石油输出国组织 D.①WTO:世界贸易组织 ②WHO:世界卫生组织 9."多头领导"是指由于组织结构设置不合理导致一个下级同时接受多个上级领导的现象。

下面俗语涉及这一现象的是

A.三个和尚没水吃 B.一朝君子一朝臣 C.狗拿耗子多管闲事 D.一山难容二虎

10.商品差价是指同一商品由于流通环节、购销地区、购销季节以及质量不同而形成的价格

差额。下面情形属于商品差价的是

A.白萝卡一元一斤,胡萝卡五角一斤 B.国产啤酒与进口啤酒销售价格不同

C.在广西买荔枝比在北京买便直得多 D.同名书籍的精装版比简装版贵五元钱

11.下面关于文史知识的表述,不正确的一项是 A.孔子是先秦思想家,其思想核心是"仁"。 B.王充是东汉思想家,代表作是《论衡》。

C.刘艇、是南朝文学理论家,代表作是《文心雕龙》。 D.王阳明是宋代理学家,他宣扬"心学"。 12.?侵权责任法》第13条规定"法律规定承担连带责任的,被侵权人有权请求部分或者全

部连带责任人承担责任"。现有甲、乙、丙三人需对丁承担连带责任。根据此规定,下面 表述中正确的一项是

A.丁只能要求甲、乙、丙三人中的一人承担责任 B.丁不能要求甲、乙、丙三人中的二人承担责任 C.丁不能要求甲、乙、丙三人共同承担责任 D.丁可以只要求甲承担责任

13.?合同法》第44条规定:"依法成立的合同,自成立时生效。"同时该法第210条规定:”自

然人之间的借款合同,自贷款人提供借款时生效。"结合此两条规定,下面合同在成立时 不能立即生效的是

A.甲公司与乙银行签订的借款合同 B.甲公司与丙信托公司签订的借款合同

C.公民丁与公民戊签订的借款合同 D.甲公司与公民丁签订的借款合同 14.光纤通讯中信号传播的主要载体是光导纤维,其内芯和外套材料不同,利用光从折射率

大的介质进入折射率小的介质时发生全反射的性质将大量信息高速传输。下面关于光导

纤维的说法,正确的是

A.内芯的折射率比外套的大,光传播时由内芯进入外套时发生全反射 B.内芯的折射率比外套的小,光传播时由内芯进入外套时发生全反射 C.内芯的折射率比外套的大,光传播时由外套进入内芯时发生全反射 D.内芯的折射率比外套的小,光传播时由外套进入内芯时发生全反射 15.风是相对于地表面的空气运动。形成风的直接原因,是气压在水平方向分布的不均匀。另 外,风还受地球自转、地形、水域等不同因素的综合影响。因此,形成风的原因不包括 A.气压梯度 B.海陆分布

C.地表摩擦力 D.地转偏向力

二、填空题 16.在下面横线处,填入上下文衔接最恰当的一段话。 在现代社会里,相信科学就是相信牢靠的一面,相信奇迹则是相信不牢靠的一面。 ________ A.可惜的是,多数人并不了解这些严谨、乏味的科学知识是大家的安全和幸福所系,对

此缺少一种慎重和敬意。时值今日,全体人类的生存,都靠科学技术来保障。一旦老

百姓不听科学的招呼,生灵涂炭的大祸就在眼前。 B.可惜的是,多数人并不了解这些严谨、乏味的科学知识是大家的安全和幸福所系,对

此缺少一种慎重和敬意。一旦老百姓不听科学的招呼,生灵涂炭的大祸就在眼前。时

值今日,全体人类的生存,都靠科学技术来保障。

C.一旦老百姓不听科学的招呼,生灵涂炭的大祸就在眼前。时值今日,全体人类的生

存,都靠科学技术来保障。可惜的是,多数人并不了解这些严谨、乏味的科学知识是

大家的安全和幸福所系,对此缺少一种慎重和敬意。

D.时值今日,全体人类的生存,都靠科学技术来保障。可惜的是,多数人并不了解这些 严谨、乏味的科学知识是大家的安全和幸福所系,对此缺少一种慎重和敬意。一旦老

百姓不听科学的招呼,生灵涂炭的大祸就在眼前。

17.在下面横线处,填入对修辞方法及其作用的准确表述。

"凡走狗,虽或为一个资本家所拳养,其实是属于所有的资本家的,所以它遇见所

有的阔人都驯良,遇见所有的穷人都狂吠。"以上文字_______,表达了强烈的憎恶之情。

A.运用比喻手法,具体生动地表现了势利小人的卑鄙 B.运用比拟手法,具体生动地表现了势利小人的卑鄙 C.运用比拟手法,形象生动地反映了资本家帮凶的本质 D.运用比喻手法,形象生动地反映了资本家帮凶的本质 18.在下面文字中的横线处,依次填入最恰当的关联词语。

中国人和日本人还是不同,______中国人和日本人的不同,在外表上很不容易看出来。

_______每一个国家的国民,都有它特别的遗传和环境,________自然就有了他的国民性。由这一点来讲,______不能理解一国的国民性,就很难欣赏一国的文学。

A.而且 再说 因此 既然 B.只是 因为 所以 如果 C.只是 再说 因此 如果 D.而且 因为 所以 既然 19.在下文横线处,依次填入最恰当的标点符号。

当今社会______人情风和关系风正污浊着人们的心灵_______金钱多少表示人情的轻重

________礼品贵贱显示关系的亲疏________ A., 。 。 ! B , 。 。 。 C., : ; 。 D., , , ! 20.在下面各句横线处,依次填入最恰当的词语。 ①外表大大咧咧的她,其实内心里深藏着不为人知的丰富_______的感情。 ②经过_______的调查研究,这家公司最终选择上海作为中国市场切入点。 ③中国四大名绣之一的苏绣,向来以其工艺精湛、针法_______闻名世界。 A.细密 细腻 细致 B.细腻 细致 细密 C.细腻 细密 细致 D.细致 细密 细腻 21.王国维在《人间词话》中把境界分为"有我之境"与´?无我之境飞他说,"有我之境,

以我现物,故物皆著我之色彩。无我之境,以物现物,故不知何者为我,何者为物。"下

面四个词句,______最具"无我之境"的特点。

A.鱼市孤烟袅寒碧,水村残叶舞愁红 B.街南绿树春绕絮,雪满游春路 C.泪眼问花花不语,乱红飞过秋千去 D.双燕归飞绕画堂,似留恋虹梁 22.在下面横线处,依次填入正确选项。

诞生几个重要的文学社团。_______的创作主张倾向于现实主义,_______主

张"为艺术而艺术",______提倡新格律诗和"国剧"运动。 A.文学研究会 创造社 新月社 B.创造杜 文学研究会 新月社 C.文学研究会 新月社 创造社 D.新月社 创造社 文学研究会

23."理论总是灰色的,而生命之树常青",歌德的这句话不直用来说明_______的道理。

A.纸上得来终觉浅,绝知此事要躬行 B.践是检验真理的唯一标准 C.理论必须联系实际 D.理论应放之四海而皆准

24.博弈思维,是指由于事情的结果取决于自己和他人的策略选择,因此要理性分析各种各

选策略及其后果的可能性,从而使受益最大化的一种思维方式。_______反映了博弈思维。

A.塞翁失马,焉知非福 B.兵来将挡,水来土掩 C.种瓜得瓜,种豆得豆 D.城门失火,殃及池鱼

25.2012年7月24日,海南省地级三沙市在东沙群岛永兴岛挂牌成立。根据中央军委

的批复,三沙_______随之成立,隶属于海南省军区。 A.警备区 B.守备区 C.军分区 D.海监局

26.汉语有一系列用于表达称呼的"令~"类词,"令嫂"一词是对_______的称呼。

A.对方的女儿 B.对方的妻子 C.自己的女儿 D.自己的妻子

27.根据《宪法》规定,全国人民代表大会审议通过《中华人民共和国合同法?,应当由全体 代表_______通过 A.一致 B.过五分之四

C.过四分之三 D.过二分之一

28.苏格拉底教学法又称为 _______ A.问答法 B.讲授法 C.发现法 D.雄辩法

29.神舟九号飞船在2012年6月18日14时左右与天宫一号实施自动空间交会对接。空间交

会对接是指两个航天器在空间轨道上会合,并在结构上连成一个整体的技术。在交会对 接过程中,追踪飞行器的飞行将首先进入_______阶段,在地面测控的支持下,追踪飞行器

经过若干次变轨机动,进入到其敏感器能捕获目标飞行器的范围。 A.近程导引 B.远程导引 C.最终逼近D.对接停靠

30.冷暖气团交界面叫锋面。在锋面移动过程中,根据冷暖气团所占的主次地位不同,可以

将锋分为冷锋、暖锋等类型。冷锋过境时多出现雨雪、大风、降温等剧烈的天气变化,

而暖锋过境时容易形成连续性降水或下雾、升温等天气状况。根据以上描述可知,"忽 如一夜春风来,千树万树梨花开"描述的是_______过境时的天气:"清明时节雨纷纷,路

上行人欲断魂"描述的是_______过境时的天气。 A.冷锋,暖锋 B.冷锋,冷锋 C.暖锋,暖锋 D.暖锋,冷锋

三、阅读理解题

(一)阅读下面短文,回答问题

所谓制艺,是指自末以来考试的文章而言。在唐时考试用诗;末时改为经义,即从四书

或五经内出一题目,由考的人作一段文章,其形式全与散文相同;到明代便有了定型:文章

的起首是破题,其次是承题,再次是起讲,后面共有八股,每两股作为一段,此平彼仄,两

两相对。下面再有一段作为结尾。这便是所谓八股文。到明末清初时候,更加多了许多限制,

不但有一定的形式,且须有一定的格调。这样,越来便越麻烦了。 为什么会有八股文这东西起来呢?据我想这与汉字是有特别关系的。汉字在世界上算是

最特别的一种,它有平仄而且有偏旁,_______可找些合适的字使之两两互对起来。例如"红花"可用"绿叶"作对,_______用"黄叶"或"青枝"等去对,_______小学生也知其不合适,因为"红花"和"绿叶",不但所代表的颜色和物件正好相对,字的平仄也是正对的,_______红绿二字还都带有"牟"旁,其它的"青枝""黄叶"等便不足这些条件了。

从前有人路过一家养马的门口,见所贴门联的一幅是"左手牵来千里马",觉得很好,

但及至看到下幅,乃是"右手牵来千里驹",又觉得很不好了。这在卖马的人只是表示他心 中的愿望,然而看门联的人则以为应当对得很精巧才成,仿佛"千"定要对"万"或"手" 定妥对"足"才是。

这样子,由对字而到门联,由门联而到挽联,而到很长的挽联,便和八股文很接近了。

中国打"灯谜"的事也是世界各国所没有的,在中由各地方各界却都很普遍。譬如"人

人尽道看花田",打四书一句."言游过矣",又如"传语报平安"打"言不必信"等等,

意思尽管是牵强附会,但倒转过来,再变化得高级一些,便成为八股文中破题的把戏,因此, 我觉得八股文之所以造成,大部分是由于民间的风气使然,并不是专因为某个皇帝特别提倡 的缘故。

关于破题有很多笑话,但虽是笑话,其作法却和正经的破题完全相同。如有人以极通俗

的话作破题解释"三十而立"说?"两当十五之年,虽有椅子板凳而不敢坐也。"另外要举

一正经的例子:题目是"子曰" 有人的破题是"匹夫而为百世师,一言而为天下法"。

从这些例子看来,便很可以明白,低级的灯谜,和高级的破题,原是同一种道理生出来 的。

"破题"之后是"承题",承题的起首必须得用一"夫"字,例如,要接着前面所举"三

十而立"的破题作下去,其承题的起首一定是"夫椅子板凳所以坐者也„„"一类的话头。

总之,作文章的人,处处都受有限制,必须得模仿当时圣贤说话的意思,又必须遵守形

式方面的种种条规。作一篇文章消磨很多的时间,作成之后却毫没价值。 八股文中的声调也是一件很主妥的成分。这大概是和中国的戏剧有关系的事。中国的歌 曲早已失传,或者现在一般妓女所唱的小妈还有些仿佛吧,然而在民间已不通行。大多数国

民的娱乐,只是在戏剧方面。现在各学校所常举行的蒋艺会欢迎会之类,在余兴一项内也大

半都是唱些旧剧,老百姓在种地的时候,或走路害怕的时候,也都好唱几句皮簧之类,由此

可见一般人对于戏剧的注意点是在于剧词的腔调方面。当我初到北京时是在光绪三十年顷,

在戏院里见有许多当时的王公们,都脸朝侧面而不朝戏台,后来才知道这是因为他们所注意

的只是唱者的音调如何,而不在于他们的表演怎样。西皮二簧甚至昆曲的词句,大半都作得

不好,不通顺,然而他们是不管那些的,正如我们听西洋戏片,多半是只管音调而不管意思

的。这在八股文内,也造成了同样的情形,只要调子好,规矩不错,有时一点意思也没有, 都可以的。

总括起来,八股文和试帖诗都一样,其来源:一为朝廷的考试,一为汉字的特别形状,

而另一则为中国的戏剧。其时代可以说自宋朝即已开始,无非到清朝才集其大成罢了。

〈节选自周作人《清代文学的反动(上)?,见《周作人散文全集? (6),广西师范大学出版社2009) 31.根据文中所述,八股文起源于 A.唐代 B.宋代 C.明代 D.清代

32.根据上下文,在第二自然段划线处依次填入最恰当的关联词语。A.于是 又 所以 何况 B.并且 若 即使 何况 C.于是 若 即使 而且 D.并且 又 所以 而且

33.作者认为"打灯谜"和八股文的破题有相通之处,是因为"打灯谜" A.难免牵强附会 B.可以变化得高级一些 C.是中国独有的 D.是对语句的设法解读

34.根据文中所述,与八股文的产生完全无关的是 A.中国的歌曲早已失传 B.某个皇帝特别提倡 C.对戏剧腔调格外关注 D.汉字分平仄有偏旁

35.通观全文,八股文之所以"作成之后却毫没价值飞是因为 A.破题曾有很多笑话 B.讲究两股平仄相对 C.写作上有许多限制 D.只重形式不重内容 (二)阅读下面短文,回答问题

亚洲银行近日发布一份名为"亚洲和太平洋地区2010年关键指标"的报告,其中宣布,

按照每日消费2-20美元的标准,亚洲"中产阶级"的人数早在2008年就已达到19亿。而

按照这一标准,中国的中产阶级人数居然达到惊人的8.17亿。被好事者呼吁良久的"橄榄型

社会居然就这么实现了。

不过,比亚洲银行名头更响的世界银行,曾经颁布过一个"贫困标准",即每人每天收

入低于1.25美元,就叫"绝对贫困"。当然"绝对贫困"并非如字面解读的那种"赤贫"状

态,而是相对于"相对贫困"而言。也就是说不管在任何国家或地区,低于这个收入水平就 是生活在贫困之中,而与他在社会中的相对住直元关。比如说在那些整体贫困的国家,即使

是其中的中等收入阶层,其实际生活也可能在"绝对贫困"标准之下;反之在那些发达国家,

即使是少数"相对贫困"人口,其实际生活水平也往往高于那些贫困国家的富裕阶层。 明乎此,亚洲银行把亚洲和中国的"中产阶级"的低限标准确定为仅仅高于"绝对贫困" 标准0.75美元,实际是看低了整个亚洲的经济发展水平一一只有在一个整体收入水平低下的地区,其"中等收入"水平,才可能以2美元为起点。判断亚洲银行看低了中国人民的收入

水乎,自有中国的数据为证一一-每人每日消费2美元,大致折算为14元人民币,每月消费则为420元人民币。这个数字不妥说远远低于北京、上海这样的大城市,甚至竟远远低于中国大多数地区的最低工资保障水平。

但是有关中国中产阶级的标准和数量,从来都是一笔糊涂账。2005年,国家统计局根据抽样调查,给过一个中产阶层的收入标准一一-年收入在6万元到50万元之间的家庭,就是当今中国的"中产样的大城市,月收入在6000元左右的人或月收入为10000元左右的家庭,就可归入"中产"。虽然上述报告给出的结论都有根有据,但还是元一例外地,受到冷嘲热讽,几乎所有被划入"中产"的白领们,都在控诉"被中产"的冤屈和真实生活的窘迫和无奈。 但笔者认为,研究者的结论与被研究者的感受之间的巨大差异,倒并不一定是研究者的阴谋,而缘于对"中产"的定义不同。在研究者看来,"中产"只是一个相对概念,那些收入处于高、低之间的阶层,就是"中等收入阶层”。而在自认“被中产”的白领心中,“中产”则有一个被美国“中产阶级”定义和描述过的模板——稳定优厚的收入、庸常但优雅的品味、保守但理性的立场„„当无数中国“中等收入”的群体,正处在追求着这种生活的焦虑之中,却被宣布已经成为“中产”,其愤懣和怀疑是可想而知的——如果我们当下的生活就是中国的中产阶层,那我何时才能过上美国的中产生活? 其实一个元情的、显见的、确定的结论,却总是被中国的中产阶层拒绝承认,那就是中 国的中产阶层,永远不可能过上美国中产阶级那样的生活。如果中国确如美国那样,变成一

个中产阶层占多数的社会则中国的中产阶层队伍确乎要有近8亿人口。而人类绝不可能再

拥有第二个地球,提供给这8亿中国中产阶层每家一栋别墅、两辆汽车。这结论不是被认为

心怀巨测的中国研究者说的,而是奥巴马说的一一-这回你总得信了吧? 所以,尽管世界已经越来越"平",尽管那个"美好"的美国就在天际线的尽头眨着鬼

魅而诱惑的眼睛,但中国的"中产"们还是不得不把饥渴的眼睛,从美国"榜样"身上移开。 中国的"中产",由中国的社会发展水平和潜在的资源禀赋、发展空间定义,却与美国的中

产阶级没有太多的相干。

(选自胡冉《难缠的"中产吟,?北京青年报?, 2010年9月5日) 36.文章说有人认为某些研究者"心怀臣测",其原因不包括 A.故意夸大了中国中产阶层的数量

B.未能关注中等收入群体追求中产的焦虑

C.无视被划入中产的白领真实生活的窘迫和无奈 D.由于缺乏客观依据才得到一笔糊涂账

37.根据文章的内容,在中国的研究者看来,中国的"中产阶层"不是指 A.收入处在中国高、低收入之间的人群 B.至少拥有一栋别墅和两辆汽车的家庭 C.月收入6000元到10000元之间的家庭 D.年收入在6万元到50万元之间的家庭

38.下面词语,与"尽管世界己经越来越‘平,"一句中"平"的意思最接近的是

A.平和 B.平衡 C.平均 D.平稳

39.根据作者的观点,"中产阶级(阶层)"是依据_______划分的阶层。A.相对贫富标准 B.绝对贫富标准 C.美国模式标准 D.世界统一标准

40.以下对本文主旨的理解,正确的一项是

A.在现代社会中,"中产阶级"是社会的主要群体,要大力建设"橄榄型"社会

B.在中国,44中产阶级"现在不是主要群体,也永远不可能成为社会的主要群体

C."中产阶级"是有统一标准的概念,不因国家发展水平的不同而制定不同标准

D.中国要按美国模式产生"中产阶级",既不合中国的实际,也根本不可能实现 (三)阅读下面短文,回答问题

偏见可以说是思想的放假。它是没有思想的人的家常日用,而是有思想的人的星期日娱乐。假如我们不能怀挟偏见,随时随地必须得客观公平、正经严肃,那就像造屋只有客厅 没有卧立,又好比在浴室里照镜子还得做出摄影机头前的姿态。魔鬼在但丁《地狱篇》第二

十七句中自称."敝魔生平最好讲理。"可见地狱之设,正为此草;人生在世,言动专求合

理,大可不必。当然,所谓正道公理压根儿也是偏见。依照生理学常识,人心位直,并不正

中,有点偏侧,并且时髦得很,偏倾于左。古人称偏僻之道为"左道,,,颇有科学根据。不

过,话虽如此说,有许多意见还不失禅宗所谓"偏中正",例如学术理论之类。只有人生边

上的随笔、热恋片的町等等,那才是老老实实、痛痛快快的一偏之见。世界太广漠了,我

们圆睁两眼,平视正视,视野还是偏狭得可怜。狗注视着肉骨头时,何尝顾到旁边还有码呢? 三于通常所谓偏见,只好比打靶的瞄准,川眼来看。但是,也有人´以为这倒是瞄中事物

且心的看法。譬如说,柏拉图为人类下定义云:"人者,元羽毛之两足动物也。"可谓客观

极了!但是按照来时铁斯《哲人言行录》六卷二章所载,偏有人拿着一只拔了毛的鸡向柏拉

图去质问。偏激二字,本来相连;我们别有所激,见解当然会另有所偏。假使我们说"人

类是不拘日夜,不问寒暑,发出声音的动物。"那又何妨? 禽喘于春,萤啼于秋,坟作雷于夏,夜则虫醒而鸟睡,风雨并不夭夭有,元来人犬不吠,

不下蛋鸡不报。惟有人用语言,用动作,用机械,随时随地做,出声音。就是独处一室,与

酬答的时候,他可以开留声机,听无线电,甚至睡眠时还发f±:似雷的鼻息。语言当然不就是声音;但是在不中听,不愿听,或着隔着墙壁和距离听不真的语言里,文字都丧失了圭角和轮廓,变成一固忽涨忽缩的喧闹,跟鸡鸣文吠同样缺乏意义,这就是所谓人籁!断送了睡眠,震断了思想,培养了神经衰弱。

这个世界毕竟是人类主宰管领的。人的声音胜过一切。聚合了大自然的万千喉舌,抵不

上两个人同时说话的喧哗,至少从第三者的耳朵听来。人箱是寂静的致命伤,天籁是能和寂

静溶为一片的。每日东方乍白,我们梦已回而困永醒,会听到无数禽声,向早晨打招呼。那

时夜未全消,寂静还逗留着,来庇荫未找清的睡梦。数不清的麻斗室的呜噪,琐碎得像要啄破了这个寂静:乌鸪的声音清利像把剪刀,老藕鸟的声音滞涩而有刺像把锯子,都一声两声地向寂静来试锋口。但是寂静似乎太厚实了,又似乎太流动了,太宫于弹性了,给禽鸟啼破的浮面,立刻就填满。雄鸡引吭悠扬的报晓,也并未在寂静上划下一道卢迹。慢慢地,我们忘了鸟喘是在破坏寂静:似乎寂静已将鸟语吸收消化,变成一种有声音的寂静。此时只要有邻家小儿的啼哭,楼上睡人的咳嗽,或墙外早行者的脚步声,寂静就像宿雾见了朝阳,破裂分散得干净。人籁己起,人事复始,你休想更有安顿。在史l到身倦,或苦思冥想时,忽闻人籁嘈杂,最博爱的人道主义者也许有时杀心顿起,恨不能天口以博耳根清净。禽兽风涛等一切天赣能和寂静相安相得,善于体物的古诗人早已悟到。《诗经?:"萧萧马鸣,悠悠祷施",

下文就说明"有闻元声";可见马嘶而元人喊,不会产生喧闹。?颜氏家句II?也指出王籍名

句"埠嗓林愈静,鸟鸣山史幽",就是"有闻无声"的感觉;虫鸟鸣噪,反添静境。雪莱诗

里,描写啄木鸟,也说鸟´系山史幽。柯律立治《风瑟》诗云."海声远且幽,似告我以静。" 假使这个海是人海,诗人非耳聋头痛不可。所以我们常把"鸦呜崔嗓"来比人声喧哗,还是

对人类存三分回护的曲笔。常将一群妇女的说笑声比于"莺啼燕语",那简直是对于禽类的 侮辱了。

闹与热,静与冷,都有连带关系;所以在_______的地狱里,太阳也给人以_______之感。人声_______,冷屋会变成热锅,使人通身_______。叔本华《哲学小品》第二百七十八节中说,思想家应当耳聋,大有道理。因为耳朵不聋,必闻声音,声音热闹,头脑就很难保持冷静,思想不会公平,只能把偏见来代替。那时候,你忘掉了你自己也是会闸的动物,也曾嚷嚷以致隔壁的人不能思想和睡眠,你更顾不得旁人在说你偏见太深,你又添了一种偏见,又在人生边上注了一笔。

(节选自钱钟书《一个偏见?,见《写在人生边上)),三联书店2002) 41.下面符合第一段文意的观点是

A.偏见是思想的副产品 B.没有思想的人没有偏见 C.学术理论难免带有偏见 D有思想的人不能怀挟偏见

42.文中论说"人类是不拘日夜,不问寒暑,发出声音的动物"的用意在于 A.比较"人籁"和"天籁",抨击人类生活的不自然 B.以"人籁"的嘈杂反衬"天籁"的生动,讽刺人性 C.借以对"天籁"和"人籁"进行褒贬取舍 D.借以说明"别有所激,见有所偏"的道理

43.选择恰当的词语,依次填入第四段的横线处 A.阴惨寂寥喧杂烦躁 B.寂寥阴惨喧杂烦躁 C.阴惨喧杂寂寥烦躁 D.寂寥阴惨烦躁喧杂

44.作者在文中常常引用他人的话来表达意见,这样做的主要作用在于 A.使文章更加通}I顶 B.使文章更有文采 C.使文章更有说服力 D.使文章更加艰深

45.下面对文章内容的理解和概括,不恰当的一项是 A.人生在世,言动不必专求合理

B."人籁"具有唤起人类、激发思想、振奋精神之效 C."天籁"能溶于寂静的原因在听者"有闻无声" D.保持冷静是防止思想产生偏见的条件之一 (四)阅读下面短文,回答问题

在4月22日第43个世界地球日到来前,?科学》发表的一篇文章,把人们关注气候变

化与二氧化碳正在排放的视角从大气带到了海洋。这篇文章宣称,由来自不同大学的21位研究人员组成的科研小姐,经过检测和评估地质记录后,得出结论:目前的海洋酸化速度是3亿年来的最高值。 海洋酸化是指由于吸收大气中过量的二氧化吸导致海水逐渐变酸的过程。人类活动向

大气释放的二氧化钱,以每小时100万吨以上的速率被海洋吸收,在吸收过程中,二氧化碳

与水反应释放出氢离子,使得海水的pH值下降。海水应为弱域性,海洋表层水的pH值约为8.2。但到2012年,海水表层pH值降低了0.1。海水酸性的增加,会改变海水的种种化学平衡, 使多种海洋生物乃至生态系统面临巨大威胁。因此,除了全球变暖,海洋酸化被称为与二氧化破排放相关的另一重大环境问题。

一份来自夏威夷附近海域20年的数据显示,工业革命以来,海水表层pH佳从1960年的8.15下降到8.05,这表示,海水中氢离子浓度增加了30。 不仅如此,海洋酸化的速度也越来越快。最近一项研究表明,海表吸收二氧化碳的速率

及其所导致的海洋酸化速率比两万年前的末次冰期快了近100倍,而末次冰期被认为是最近

一次的二氧化碳急剧上升期。根据政府间气候变化专门委员会CIpCC)的预测,如按照目前

二氧化破排放量的水平进行,本世纪末,海水pH值将下降至7.8左右。 研究表明,在二氧化碳浓度加倍以后,大多数钙化生物的钙化速率均大幅下降。钙化速 率的下降,不仅影响到了浮游性钙化生物,如颗石藻等向底层海洋的破输送,还会影响到钙 化动物的生长和发育。同时,珊湖藻以及造礁珊湖种类在加倍的二氧化项条件下,其钙化速 率平均下降30。 多项研究表明,海洋酸化还可能通过食物链,造成原本不同种间的配子受精成功并形成

杂交种,造成种质混乱,影响物种间的相王作用及生态系统的稳定性。例如,某些浮涝动物

在喂食酸化海水中生长的浮将植物后,繁殖率显著下降。 有研究者在模拟实验中发现,严重酸化的海水中,小丑鱼幼鱼将失去听力、视力、嗅觉,

元法发现敌害,也丧失了相应的逃逸和生存能力。对甲壳类、贝类、鱼类及林皮动物等海水 养殖生物的研究表明,海洋酸化会显著地影响到幼体发育,降低成体的钙化率和呼吸活动,

改变机体能量代谢方式,干扰感知和运动行为,抑制免疫防御系统的活性,引起生物体代谢

异常、生长缓慢甚至死亡。已经有研究表明,海洋酸化是一种生理胁迫,会使得藤壶的成活

率显著下降,二氧化破浓度升高同样会导致海胆的尺寸和重量均明显变小。

由于人类从未经历过这种变化,并且不同种类的海洋生物对酸化引起的海水化学变化敏 感性不同,以至于元法确定海洋酸化的生物学效应,也就无法预测未来可能发生的变化。

(节选自游雪晴《越来越酸的海洋越来越迷的未来?,中国科技网, 2012年4月22日) 46.下面关于海洋酸化的理解,正确的一项是 A.海水一旦吸收空气中的二氧化碳,即导致海洋酸、化 B.海水应为弱碱性,海洋酸化是指海水呈弱酸性或酸性 C.二氧化碳与水反应释放出氢离子,使海水的pH值升高 D.洋酸化使海水酸性增加,从而影响海水中的化学平衡 47.下面关于海洋酸化速率的描述,正确的一项是

A.夏威夷附近海域数据显示, 20年来海水表层pH值下降了0.1 B.海洋酸化速率越来越快,己达到末次冰期的100倍 C.目前,海水中的氢离子浓度是1960年的1.3倍 D.到本世纪末,海水的pH值一定会在7.8左右 48.海洋酸化可能造成的后果是

A.某些浮游动物繁殖能力下降 B.成鱼依然具备较好的逃逸能力 C.海洋生物代谢能力提高 D.贝类和棘皮动物御敌能力不变

49.为减少二氧化碳排放,降低海洋酸化速度,下面措施不正确的是 A.购物循环使用环保袋 B.避免使用一次性餐具

C.使用面巾纸代替布于帕 D.提倡自行车、步行出行 50.下面表述符合文意的一项是

A.所有的浮游动物在喂食酸化海水中生长的浮游植物后,繁殖率都会显著下降

B.珊珊藻以及造礁珊珊在加倍的二氧化碳条件下,其钙化速率将会平均下降30 C.目前海洋酸化速度,是3亿年前的130 D.人类从未经历过海洋急剧酸化的过程,但己可以用科学预测未来变化 第一部分 语言表达能力测试答案 01C,02B,03C,04C,05B, 06D,07B,08A,09A,10C; 11D,12D,13C,14A,15C, 16D,17C,18B,19C,20B; 21B,22A,23D,24D,25A, 26A,27D,28A,29B,30A; 31C,32C,33B,34A,35D, 36D,37B,38C,39A,40D; 41A,42D,43A,44C,45B, 46D,47B,48A,49C,50B.

gct英语范文
《gct英语范文.doc》
将本文的Word文档下载到电脑,方便编辑。
推荐度:
点击下载文档
相关专题
点击下载本文文档